Annotation of db/baza/rusvuz17.txt, revision 1.3

1.1       rubashki    1: Чемпионат:
                      2: Открытый чемпионат вузов России по ЧГК. 2016/17
                      3: 
                      4: Дата:
                      5: 00-000-2016
                      6: 
                      7: Тур:
                      8: Первый игровой день. 1 тур
                      9: 
                     10: Дата:
                     11: 30-Sep-2016
                     12: 
                     13: Редактор:
                     14: Илья Иванов (Пермь)
                     15: 
                     16: Инфо:
                     17: Редактор благодарит за помощь в подготовке пакета Виктора Абдураманова
                     18: (Березники), Арсэна Атнагулова (Уфа), Олега Белослудцева, Константина
                     19: Костенко, Вадима Опутина, Александра Пономарёва, Александра Шишкина,
                     20: Юлию Шучалову и Дениса Яруллина (все - Пермь), Александра Быкова и Ирину
                     21: Пинегину (оба - Киров), Андрея Ефремова (Могилев), Андрея Кокуленко
                     22: (Омск), Александра Мудрого (Черновцы), Константина Науменко (Киев),
                     23: Ксению Рудич (Москва), Константина Сахарова (Ивантеевка), Ульяну
1.2       rubashki   24: Фабричнину (Санкт-Петербург), а также Дмитрия Башука и команду "От винта
1.1       rubashki   25: - Братья по фазе" (Харьков).
                     26: 
                     27: Вопрос 1:
                     28: В пражском ресторане "Vytopna" [вЫтопна] процесс доставки пива
                     29: механизирован: клиенты получают свое пиво, когда происходит ЭТО. Кто
                     30: создал "ЭТО" в конце XIX века?
                     31: 
                     32: Ответ:
                     33: [Братья] Люмьер.
                     34: 
                     35: Комментарий:
                     36: За доставку пива в ресторане отвечает миниатюрная железная дорога -
                     37: поезд с бокалом подъезжает к посетителю, и тот может забрать стакан.
                     38: "Прибытие поезда" - знаменитый фильм братьев Люмьер.
                     39: 
                     40: Источник:
                     41:    1. http://luxlux.net/prazhskiy-restoran-vytopna-poezd-vmesto-ofitsianta-35437/
                     42:    2. http://ru.wikipedia.org/wiki/Прибытие_поезда_на_вокзал_Ла-Сьота
                     43: 
                     44: Автор:
                     45: Игорь Демьянцев (Гомель)
                     46: 
                     47: Вопрос 2:
                     48: В американском комедийном мультсериале сотрудники одной организации
                     49: используют фургон, предлагающий свежие буррито. Назовите эту
                     50: организацию.
                     51: 
                     52: Ответ:
                     53: ФБР.
                     54: 
                     55: Зачет:
                     56: FBI; Федеральное Бюро Расследований; Federal Bureau of Investigation.
                     57: 
                     58: Комментарий:
                     59: Спецслужбы часто используют фургоны для слежки. В мультсериале агенты
                     60: ФБР (по-английски - FBI) умудрились использовать фургон с надписью
                     61: "Fresh Burritos Instantly" [фреш буррИтос Инстантли] ("Свежие буррито
                     62: немедленно"), сокращенно - FBI.
                     63: 
                     64: Источник:
                     65: Мультсериал "Симпсоны", s25e01.
                     66: 
                     67: Автор:
                     68: Илья Иванов (Пермь)
                     69: 
                     70: Вопрос 3:
                     71: В 1894 году астроном Эндрю Дуглас предложил построить обсерваторию в
                     72: городке Флагстафф в Аризоне. Рассказывают, что в конце жизни Дуглас
                     73: признался, что во Флагстаффе его интересовали только луны. В одном из
                     74: слов предыдущего предложения мы пропустили две буквы. Напишите это слово
                     75: в исходном варианте.
                     76: 
                     77: Ответ:
                     78: Салуны.
                     79: 
                     80: Комментарий:
                     81: Территория славилась своими салунами - Аризона была частью дикого
                     82: Запада, а дело было в конце XIX века. Вот такой вот "научный" критерий
                     83: выбора для обсерватории.
                     84: 
                     85: Источник:
                     86:    1. http://ru.wikipedia.org/wiki/Обсерватория_Лоуэлла
                     87:    2. http://www.peoples.ru/science/astronomy/andrew_ellicott_douglas/
                     88: 
                     89: Автор:
                     90: Илья Иванов (Пермь)
                     91: 
                     92: Вопрос 4:
                     93: По несколько парадоксальному утверждению комментатора, эксцентричный
                     94: Фабьен Бартез иногда устраивал в воротах ЕЕ. Назовите ЕЕ словом с
                     95: удвоенной согласной.
                     96: 
                     97: Ответ:
                     98: Буффонада.
                     99: 
                    100: Комментарий:
                    101: Бартез в воротах постоянно "чудил". Джанлуиджи Буффон - другой
                    102: легендарный футбольный голкипер.
                    103: 
                    104: Источник:
                    105: Трансляция финала чемпионата мира по футболу 1998 года Франция -
                    106: Бразилия на телеканале "Матч-ТВ", 08.07.2016 г.
                    107: 
                    108: Автор:
                    109: Илья Иванов (Пермь)
                    110: 
                    111: Вопрос 5:
                    112: [Ведущему: обязательно произнести название игры полностью!]
                    113:    В начале игры "Шерлок Холмс: Преступления и наказания" главный герой,
                    114: стреляя по вазам, приветствует инспектора Лестрейда, чем изрядно
                    115: удивляет последнего. Какие три слова мы пропустили в этом вопросе?
                    116: 
                    117: Ответ:
                    118: С завязанными глазами.
                    119: 
                    120: Зачет:
                    121: С закрытыми глазами; другие синонимичные ответы из трех слов.
                    122: 
                    123: Комментарий:
                    124: Несмотря на завязанные глаза, Холмс определяет, что в комнату вошел
                    125: именно инспектор Лестрейд, - главным образом, благодаря бряцанью
                    126: наручников, звуку шагов и тому, что "скрипучая" ступенька не скрипнула
                    127: (значит, гость бывает в доме часто).
                    128: 
                    129: Источник:
                    130: Игра "Sherlock Holmes: Crimes and Punishments".
                    131: 
                    132: Автор:
                    133: Илья Иванов (Пермь)
                    134: 
                    135: Вопрос 6:
                    136: Герой фильма "Космос как предчувствие" знакомится в поезде с молодым
                    137: летчиком Юрием. Когда тот направляется к выходу, наблюдательный герой
                    138: окликает его и указывает на один из НИХ. Назовите ИХ.
                    139: 
                    140: Ответ:
                    141: Шнурки.
                    142: 
                    143: Комментарий:
                    144: Создатели фильма явно решили сделать отсылку к известной истории, когда
                    145: у Гагарина, шедшего по аэродрому Внуково к правительственной трибуне,
                    146: развязался шнурок. Впрочем, существует версия, что это были подтяжки для
                    147: носков - однако в фильме, конечно, фигурировали именно шнурки.
                    148: 
                    149: Источник:
                    150:    1. Х/ф "Космос как предчувствие" (2005), реж. Алексей Учитель.
                    151:    2. http://www.retroportal.ru/gagarin_14_04_1961.shtml
                    152: 
                    153: Автор:
                    154: Илья Иванов (Пермь)
                    155: 
                    156: Вопрос 7:
                    157: Герой французской комедии любезно отдает молодой немке солонку и
                    158: перечницу. Недовольная жена язвительно спрашивает, почему же он не
                    159: решился предложить девушке арию. В одном из слов предыдущего предложения
                    160: мы пропустили шесть букв. Напишите это слово в исходном варианте.
                    161: 
                    162: Ответ:
                    163: Лотарингию.
                    164: 
                    165: Комментарий:
                    166: Лотарингия - знаменитая провинция, которая (вместе с Эльзасом) долгое
                    167: время была причиной ожесточенных сражений и споров между Францией и
                    168: Германией.
                    169: 
                    170: Источник:
                    171: Х/ф "Каникулы маленького Николя" (2014), реж. Лоран Тирар.
                    172: 
                    173: Автор:
                    174: Илья Иванов (Пермь)
                    175: 
                    176: Вопрос 8:
                    177: В уже упомянутой нами игре на стене в комнате Шерлока Холмса можно
                    178: увидеть ИКС, написанный в 1872 году. Назовите ИКС двумя словами.
                    179: 
                    180: Ответ:
                    181: Портрет Достоевского.
                    182: 
                    183: Комментарий:
                    184: Игра называется "Шерлок Холмс: Преступления и наказания", что является
                    185: отсылкой к знаменитому роману Федора Михайловича.
                    186: 
                    187: Источник:
                    188:    1. Игра "Sherlock Holmes: Crimes and Punishments".
                    189:    2. https://www.wikiart.org/ru/vasiliy-perov/portret-pisatelya-fedora-mikhaylovicha-dostoevskogo-1872
                    190: 
                    191: Автор:
                    192: Илья Иванов (Пермь)
                    193: 
                    194: Вопрос 9:
                    195: На карикатуре Жана Эффеля Бог, прогуливающийся среди хаоса, замечает,
                    196: что мир - это ИКС. Одним из лучших ИКСОВ 2015 года был признан
                    197: "нипричёмыш". Какое слово греческого происхождения мы заменили ИКСОМ?
                    198: 
                    199: Ответ:
                    200: Неологизм.
                    201: 
                    202: Комментарий:
                    203: На карикатуре, посвященной сотворению мира, Бог достаточно справедливо
                    204: называет создаваемый им мир неологизмом. "Нипричёмыш" - скучный, тихий
                    205: человек, всегда держащийся подальше от любых волнений, - был признан
                    206: Экспертным советом при Центре творческого развития русского языка одним
                    207: из лучших неологизмов 2015 года.
                    208: 
                    209: Источник:
                    210:    1. Ж. Эффель. Сотворение мира. - М.: Изобразительное искусство, 1989.
                    211:    2. https://snob.ru/profile/27356/blog/101993
                    212: 
                    213: Автор:
                    214: Виктор Абдураманов (Березники)
                    215: 
                    216: Вопрос 10:
                    217: По анекдотической версии Александра Экстера, жена однажды увидела
                    218: картину мужа из цикла работ о женской бане и пришла в ярость. Какое
                    219: произведение, согласно этой версии, было создано в результате?
                    220: 
                    221: Ответ:
                    222: "Черный квадрат".
                    223: 
                    224: Комментарий:
                    225: И в ярости замазала полотно черной краской. Так Малевич и придумал свой
                    226: "Черный квадрат" (во что, конечно, верится с трудом).
                    227: 
                    228: Источник:
                    229: http://www.vokrugsveta.ru/article/205155/
                    230: 
                    231: Автор:
                    232: Илья Иванов (Пермь)
                    233: 
                    234: Вопрос 11:
                    235: Одна из шведских церквей недавно вывесила объявление с напоминанием, что
                    236: по всё еще действующему закону 1280 года никто не может быть взят в плен
                    237: в церкви. Отдельно подчеркнуто, что под защиту закона подпадают и ОНИ.
                    238: Назовите ИХ одним словом.
                    239: 
                    240: Ответ:
                    241: Покемоны.
                    242: 
                    243: Комментарий:
                    244: Судя по всему, любители поохотиться на покемонов делали это и в церкви.
                    245: 
                    246: Источник:
                    247: http://www.diary.ru/~Logovojuni/p209907950.htm
                    248: 
                    249: Автор:
                    250: Константин Костенко (Гремячинск - Пермь)
                    251: 
                    252: Вопрос 12:
                    253: Опера ДжАкомо Пуччини "ТОска" заканчивается прыжком главной героини с
                    254: высокой башни. По легенде, конец одной из постановок "ТОски" привел
                    255: публику в восторг. Причиной этого стало использование ЕГО. Назовите ЕГО
                    256: одним словом.
                    257: 
                    258: Ответ:
                    259: Батут.
                    260: 
                    261: Комментарий:
                    262: Исполнительница главной роли спрыгнула из декорационной башни, однако
                    263: вместо привычных матрасов кто-то додумался установить батут. В итоге
                    264: певица три или четыре раза подпрыгнула на нем, что не осталось
                    265: незамеченным публикой.
                    266: 
                    267: Источник:
                    268: Э. Ланди. Тайная жизнь великих композиторов.
                    269: http://www.flibusta.is/b/390922/read
                    270: 
                    271: Автор:
                    272: Илья Иванов (Пермь)
                    273: 
                    274: Тур:
                    275: Первый игровой день. 2 тур
                    276: 
                    277: Дата:
                    278: 30-Sep-2016
                    279: 
                    280: Редактор:
                    281: Андрей Ефремов (Могилев)
                    282: 
                    283: Инфо:
                    284: Редактор благодарит за помощь в тестировании Валерия Семёнова, Сергея
                    285: Дубелевича и команду "Солянка" (все - Минск), команду "От винта - Братья
                    286: по фазе" (Харьков) и Лидию Иоффе (Хайфа).
                    287: 
                    288: Вопрос 1:
                    289: В научно-популярном фильме параллельная эволюция вирусов и человеческого
                    290: организма сравнивается с НЕЙ. Назовите ЕЕ двумя словами, начинающимися
                    291: на соседние буквы алфавита.
                    292: 
                    293: Ответ:
                    294: Гонка вооружений.
                    295: 
                    296: Комментарий:
                    297: Вирусы и человеческие клетки на протяжении тысячелетий вынуждены были
                    298: изобретать всё новые и новые механизмы атаки и защиты, обмана и его
                    299: распознавания.
                    300: 
                    301: Источник:
                    302: Эфир телеканала "Наука 2.0", сериал "Тайная вселенная", серия
                    303: "Путешествие внутрь клетки", эфир от 11.02.2015 г.
                    304: 
                    305: Автор:
                    306: Андрей Ефремов (Могилев)
                    307: 
                    308: Вопрос 2:
                    309: Автор исторического романа сравнил ЕЕ с заточённой в монастырь царицей,
                    310: которую ветреный и капризный властелин променял на холодную,
                    311: змеиноглазую разлучницу. Назовите ЕЕ одним словом.
                    312: 
                    313: Ответ:
                    314: Москва.
                    315: 
                    316: Комментарий:
                    317: Как известно, в XVIII веке Петр I перенес столицу в Санкт-Петербург.
                    318: 
                    319: Источник:
                    320: Б. Акунин. Любовница смерти. http://www.flibusta.is/b/500333/read
                    321: 
                    322: Автор:
                    323: Андрей Ефремов (Могилев)
                    324: 
                    325: Вопрос 3:
                    326: В статье об ИКСЕ упоминаются коронарные артерии. В произведении XIV века
                    327: обитатели ИКСА стараются прикрыть один бок другим. Назовите ИКС двумя
                    328: словами.
                    329: 
                    330: Ответ:
                    331: Третий круг.
                    332: 
                    333: Комментарий:
                    334: Третий круг кровообращения обслуживает само сердце. Во втором
                    335: предложении речь идет о третьем круге ада в описании Данте, в который
                    336: помещены чревоугодники, стыдящиеся своей полноты. Вопрос в туре идет
                    337: третьим - это дополнительная подсказка.
                    338: 
                    339: Источник:
                    340:    1. http://www.science-education.ru/34-1342
                    341:    2. https://otvet.mail.ru/question/47811228
                    342: 
                    343: Автор:
                    344: Андрей Ефремов (Могилев)
                    345: 
                    346: Вопрос 4:
                    347: На карикатуре, действие которой происходит весной, ПЕРВЫЕ испуганно
                    348: смотрят на появляющиеся из земли ВТОРЫЕ. Какие однокоренные слова мы
                    349: обозначили как "ПЕРВЫЕ" и "ВТОРЫЕ"?
                    350: 
                    351: Ответ:
                    352: Снеговики, подснежники.
                    353: 
                    354: Источник:
                    355: http://pikabu.ru/story/predvestniki_smerti_1944655
                    356: 
                    357: Автор:
                    358: Андрей Ефремов (Могилев)
                    359: 
                    360: Вопрос 5:
                    361: В одном из стихотворений Владимир Маяковский удивляется: неужели "зуб
                    362: революций ступился о НИХ"? Какая денежная единица получила свое название
                    363: от НЕЕ?
                    364: 
                    365: Ответ:
                    366: Крона.
                    367: 
                    368: Комментарий:
                    369: ОНА - это корона. Как известно, прокатившаяся по Европе волна революций
                    370: способствовала падению не одной монархии.
                    371: 
                    372: Источник:
                    373:    1. В.В. Маяковский. Радоваться рано.
                    374: http://www.feb-web.ru/feb/mayakovsky/texts/ms0/ms2/ms2-016-.htm
                    375:    2. http://ru.wikipedia.org/wiki/Крона_(денежная_единица)
                    376: 
                    377: Автор:
                    378: Андрей Ефремов (Могилев)
                    379: 
                    380: Вопрос 6:
                    381: В вопросе словом "ИКС" заменено другое слово.
                    382:    Однажды в компьютерной игре "Герои меча и магии - 5" автору вопроса
                    383: встретился ИКС дьяволят. Назовите ИКС, созданный в 1831 году королевским
                    384: приказом.
                    385: 
                    386: Ответ:
                    387: [Французский] Иностранный [легион].
                    388: 
                    389: Комментарий:
                    390: Словосочетание "легион дьяволят" отсылает к цитате из Евангелия "Имя мне
                    391: - легион". Иностранный легион был создан по приказу Луи-Филиппа I, а
                    392: Почетный легион - еще при Наполеоне I.
                    393: 
                    394: Источник:
                    395:    1. ЛОАВ (пользовательская карта "Живее всех живых").
                    396:    2. http://ru.wikipedia.org/wiki/Имя_мне_%E2%80%94_легион
                    397:    3. http://ru.wikipedia.org/wiki/Французский_Иностранный_легион
                    398: 
                    399: Автор:
                    400: Андрей Ефремов (Могилев)
                    401: 
                    402: Вопрос 7:
                    403: В сентябре 1957 года сразу после аварии на химкомбинате "Маяк" в
                    404: челябинских газетах появились сообщения о НЕМ. ОНО бывает двух видов,
                    405: которые на самом деле вовсе не симметричны. Назовите ЕГО двумя словами,
                    406: начинающимися на разные буквы.
                    407: 
                    408: Ответ:
                    409: Полярное сияние.
                    410: 
                    411: Комментарий:
                    412: Советские газеты пытались дезинформировать население, выдав техногенную
                    413: катастрофу за природное явление. До недавнего времени считалось, что
                    414: полярные сияния в северном и южном полушарии являются симметричными.
                    415: Однако одновременное наблюдение полярного сияния в мае 2001 года из
                    416: космоса со стороны северного и южного полюсов показало, что северное и
                    417: южное сияния существенно отличаются друг от друга.
                    418: 
                    419: Источник:
                    420:    1. http://ru.wikipedia.org/wiki/Кыштымская_авария
                    421:    2. http://ru.wikipedia.org/wiki/Полярное_сияние
                    422: 
                    423: Автор:
                    424: Андрей Ефремов (Могилев)
                    425: 
                    426: Вопрос 8:
                    427: Описывая шумную толпу в лондонском порту, Филип Депуа пишет, что
                    428: невидимая ОНА поднималась до самого неба. Назовите ЕЕ двумя словами.
                    429: 
                    430: Ответ:
                    431: Вавилонская башня.
                    432: 
                    433: Комментарий:
                    434: Писатель упоминает "разноязыкий гомон". Слово "толпа" в вопросе -
                    435: подсказка.
                    436: 
                    437: Источник:
                    438: Ф. Депуа. Тайна короля Якова. http://www.flibusta.is/b/254925/read
                    439: 
                    440: Автор:
                    441: Андрей Ефремов (Могилев)
                    442: 
                    443: Вопрос 9:
                    444: При создании ИХ в резиденции сёгуна гвозди располагали в виде
                    445: перевернутой буквы V [ви] для имитации щебета различных птиц, по
                    446: которому можно было достаточно точно определить местонахождение
                    447: посетителей. Назовите ИХ двумя словами, начинающимися на одну и ту же
                    448: букву.
                    449: 
                    450: Ответ:
                    451: Поющие полы.
                    452: 
                    453: Зачет:
                    454: Птичьи полы.
                    455: 
                    456: Комментарий:
                    457: Поющие (соловьиные) полы были созданы в замке сёгунов в Киото, в первую
                    458: очередь, для того чтобы враги не могли пробраться туда незамеченными. В
                    459: зависимости от того, голос какой именно птицы был слышен, можно было
                    460: точно определить местонахождение проникнувшего в замок.
                    461: 
                    462: Источник:
                    463: https://historyporn.dirty.ru/poiushchie-poly-na-strazhe-siogunov-540719/
                    464: 
                    465: Автор:
                    466: Андрей Ефремов (Могилев)
                    467: 
                    468: Вопрос 10:
                    469: Престарелый герой рассказа Стивена Кинга сравнивает свое тело с
                    470: разрушенным ИКСОМ. Самый высокий ИКС был создан в Майами. Назовите ИКС.
                    471: 
                    472: Ответ:
                    473: Замок из песка.
                    474: 
                    475: Зачет:
                    476: Синонимичные ответы.
                    477: 
                    478: Комментарий:
                    479: Известно выражение "песок сыплется". Город Майами известен своими
                    480: пляжами. Кстати, предыдущий вопрос тоже был о зАмке.
                    481: 
                    482: Источник:
                    483:    1. С. Кинг. Человек в черном костюме.
                    484: http://www.flibusta.is/b/182301/read
                    485:    2. http://lenta.ru/news/2015/10/27/zamok/
                    486: 
                    487: Автор:
                    488: Андрей Ефремов (Могилев)
                    489: 
                    490: Вопрос 11:
                    491: Блиц.
                    492:    На одном специализированном обучающем сайте есть цикл детских
                    493: стихотворений.
                    494:    1. В одном из стихотворений ОНА "словно подлодка, стреляет прямой
                    495: наводкой". Назовите ЕЕ одним словом.
                    496:    2. В одном из стихотворений ОН предпочитает рукопашную. Назовите ЕГО
                    497: одним словом.
                    498:    3. В одном из стихотворений ОНА решает, кем стать. Назовите ЕЕ одним
                    499: словом.
                    500: 
                    501: Ответ:
                    502:    1. Ладья.
                    503:    2. Король.
                    504:    3. Пешка.
                    505: 
                    506: Комментарий:
                    507: В шахматах ладья контролирует вертикаль и горизонталь, король бьет
1.3     ! rubashki  508: только соседние клетки, а пешка, дойдя до последней горизонтали,
        !           509: превращается в одну из четырех других фигур.
1.1       rubashki  510: 
                    511: Источник:
                    512:    1. http://fenix64.com/stix-o-lade/
                    513:    2. http://fenix64.com/stix-pro-korolya-2/
                    514:    3. http://fenix64.com/stix-pro-peshku/
                    515: 
                    516: Автор:
                    517: Андрей Ефремов (Могилев)
                    518: 
                    519: Вопрос 12:
                    520: Решительно настроенный президент США Уильям Тафт считал, что "ЭТО" может
                    521: успокоить забастовку рабочих. Обычно за один сеанс ЭТОГО устанавливают
                    522: от четырех до двенадцати... Кого?
                    523: 
                    524: Ответ:
                    525: Пиявок.
                    526: 
                    527: Комментарий:
                    528: Речь идет о кровопускании. Тафт ратовал за применение силы в отношении
                    529: бастующих.
                    530: 
                    531: Источник:
                    532:    1. http://ru.wikipedia.org/wiki/Тафт,_Уильям_Говард
                    533:    2. http://ru.wikipedia.org/wiki/Кровопускание
                    534: 
                    535: Автор:
                    536: Андрей Ефремов (Могилев)
                    537: 
                    538: Тур:
                    539: Первый игровой день. 3 тур
                    540: 
                    541: Дата:
                    542: 30-Sep-2016
                    543: 
                    544: Редактор:
                    545: Дмитрий Башук (Харьков)
                    546: 
                    547: Инфо:
                    548: Редактор благодарит за помощь в подготовке вопросов команду "От винта -
                    549: Братья по фазе" (Харьков).
                    550: 
                    551: Вопрос 1:
                    552: Шутники утверждают, что для грузчиков характерно раздельное питание.
                    553: Когда они разгружают картошку, то едят картошку, когда разгружают сыр -
                    554: едят сыр. А когда разгружают кирпич, то это - ОН. Назовите ЕГО двумя
                    555: словами.
                    556: 
                    557: Ответ:
                    558: Разгрузочный день.
                    559: 
                    560: Комментарий:
                    561: "Разгрузочный день", как и "раздельное питание", - устойчивое понятие в
                    562: диетологии.
                    563: 
                    564: Источник:
                    565: "Арт-Мозаика", 2016, N 34.
                    566: 
                    567: Автор:
                    568: Дмитрий Башук (Харьков)
                    569: 
                    570: Вопрос 2:
                    571: Одну из актрис Фаина Раневская причислила к отряду молеобразных и
                    572: объяснила это тем, что у той все мысли только об АЛЬФАХ. Стихотворение
                    573: Семена Кирсанова об АЛЬФЕ заканчивается словами "... чтоб не мог мороз
                    574: ущипнуть тебя". Назовите АЛЬФУ.
                    575: 
                    576: Ответ:
                    577: Шуба.
                    578: 
                    579: Источник:
                    580:    1. Ф.Г. Раневская. Мудрые остроты Раневской.
                    581: http://www.flibusta.is/b/380824/read
                    582:    2. С.И. Кирсанов. Нащот шубы.
                    583: http://www.flibusta.is/b/343461/read#t21
                    584: 
                    585: Автор:
                    586: Дмитрий Башук (Харьков)
                    587: 
                    588: Вопрос 3:
                    589: Для десятилетнего Нила Сети, исполнившего роль Маугли в недавнем фильме
                    590: "Книга джунглей", костюмеры изготовили двадцать вариантов ИХ, которые
                    591: меняли в зависимости от эпизода. Назовите ИХ двумя словами.
                    592: 
                    593: Ответ:
                    594: Набедренные повязки.
                    595: 
                    596: Комментарий:
                    597: Других костюмов Маугли и не надо. :-)
                    598: 
                    599: Источник:
                    600: "Телескоп", 2016, N 16.
                    601: 
                    602: Автор:
                    603: Дмитрий Башук (Харьков)
                    604: 
                    605: Вопрос 4:
                    606: У основания кафедры венского собора Святого Стефана есть скульптурное
                    607: изображение человека с циркулем в руке. Назовите это изображение словом,
                    608: происходящим от древнегреческого и французского слов.
                    609: 
                    610: Ответ:
                    611: Автопортрет.
                    612: 
                    613: Комментарий:
                    614: (pic: 20160953.jpg)
                    615:    Создатель кафедры каменотёс Антон Пильграм изобразил себя взирающим
                    616: на дело своих рук; слово "автопортрет" происходит от древнегреческого
                    617: "autos" и французского "portrait".
                    618: 
                    619: Источник:
                    620:    1. Е.Н. Грицак. Вена. http://www.flibusta.is/b/116707/read
                    621:    2. http://ru.wiktionary.org/wiki/автопортрет
                    622: 
                    623: Автор:
                    624: Дмитрий Башук (Харьков)
                    625: 
                    626: Вопрос 5:
                    627: [Ведущему: при чтении явно указать запятую после первого пропуска.]
                    628:    Опальный поэт из книги Артуро Переса-Реверте в одном из эпизодов так
                    629: каламбурит о себе: "Дела идут [пропуск], дела [пропуск] дышат...".
                    630: Заполните пропуски - двумя словами каждый.
                    631: 
                    632: Ответ:
                    633: "... на лад...", "... на ладан...".
                    634: 
                    635: Комментарий:
                    636: Такой вот грустный каламбур.
                    637: 
                    638: Источник:
                    639: А. Перес-Реверте. Капитан Алатристе.
                    640: http://www.flibusta.is/b/456025/read
                    641: 
                    642: Автор:
                    643: Дмитрий Башук (Харьков)
                    644: 
                    645: Вопрос 6:
                    646: Материал журнала "Вокруг света" под заголовком "Личное влияние" посвящен
                    647: ИМ. Кстати, одна из ИХ разновидностей называлась словом, однокоренным к
                    648: слову из приведенного заголовка. Назовите ИХ.
                    649: 
                    650: Ответ:
                    651: Маски.
                    652: 
                    653: Комментарий:
                    654: Металлическая маска под названием "личина" была частью военного шлема.
                    655: 
                    656: Источник:
                    657:    1. "Вокруг света", 2016, N 7.
                    658:    2. http://ru.wikipedia.org/wiki/Личина
                    659: 
                    660: Автор:
                    661: Дмитрий Башук (Харьков)
                    662: 
                    663: Вопрос 7:
                    664: Площадь ЕЕ составляет около 4152 квадратных километров и с каждым годом
                    665: увеличивается. В 1991 году румынская часть, составляющая примерно 83%
                    666: общей площади, была признана ЮНЕСКО объектом Всемирного природного
                    667: наследия. Назовите ЕЕ двумя словами, которые начинаются на одну и ту же
                    668: букву.
                    669: 
                    670: Ответ:
                    671: Дельта Дуная.
                    672: 
                    673: Комментарий:
                    674: Остальные 17% принадлежат Украине; с каждым годом береговая линия
                    675: Черного моря смещается метров на сорок вглубь моря из-за наносов.
                    676: 
                    677: Источник:
                    678: http://ru.wikipedia.org/wiki/Дельта_Дуная
                    679: 
                    680: Автор:
                    681: Александр Лисянский (Харьков)
                    682: 
                    683: Вопрос 8:
                    684: В отличие от математики, в жизни минус на минус не всегда дает плюс.
                    685: Видимо, поэтому, согласно закону города Лик-Спрингс в штате Индиана,
                    686: выпуская на улицу черных котов в определенный день, хозяева обязаны
                    687: надевать на своих питомцев колокольчики. Назовите этот день абсолютно
                    688: точно.
                    689: 
                    690: Ответ:
                    691: Пятница, 13-е.
                    692: 
                    693: Комментарий:
                    694: Авторы закона, видимо, считают, что если нельзя избежать негативного
                    695: влияния пятницы, 13-го, то хотя бы черных котов, собирающихся перейти
                    696: дорогу, горожане в этот день заблаговременно обнаружат по звуку
                    697: колокольчиков. :-)
                    698: 
                    699: Источник:
                    700: http://www.koshsps.ru/blackcat.php
                    701: 
                    702: Автор:
                    703: Дмитрий Башук (Харьков)
                    704: 
                    705: Вопрос 9:
                    706: Персонаж книги Хольма Ван Зайчика рассуждает: "Невозможно представить
                    707: себе в просторах Запретного города, скажем, велосипед... это вопиюще
                    708: несообразно и даже несколько оскорбительно - как если бы, скажем, ИКС
                    709: вдруг запел...". Чуть больше сорока пяти лет назад так и произошло.
                    710: Назовите ИКСА.
                    711: 
                    712: Ответ:
                    713: Иисус Христос.
                    714: 
                    715: Комментарий:
                    716: В 1970 году была создана рок-опера "Иисус Христос - суперзвезда".
                    717: 
                    718: Источник:
                    719:    1. Х. Ван Зайчик. Дело Судьи Ди. http://www.flibusta.is/b/73894/read
                    720:    2. http://ru.wikipedia.org/wiki/Иисус_Христос_%E2%80%94_суперзвезда
                    721: 
                    722: Автор:
                    723: Дмитрий Башук (Харьков)
                    724: 
                    725: Вопрос 10:
                    726: Одной из причин распада этой группы в апреле 1981 года Лоуренс ДжУбер
                    727: считает то, что ее лидер стал бояться концертов (цитата): "Ведь ему
                    728: приходилось бы каждые десять минут вздрагивать, ожидая, что какой-нибудь
                    729: придурок выстрелит в него из пистолета". Назовите эту группу.
                    730: 
                    731: Ответ:
                    732: "Wings".
                    733: 
                    734: Комментарий:
                    735: Джубер - гитарист этой группы. Убийство Джона Леннона, произошедшее 8
                    736: декабря 1980 года, сильно потрясло Пола Маккартни, создавшего "Wings" в
                    737: 1971 году, вскоре после распада "Битлз".
                    738: 
                    739: Источник:
                    740: http://ru.wikipedia.org/wiki/Wings
                    741: 
                    742: Автор:
                    743: Дмитрий Башук (Харьков)
                    744: 
                    745: Вопрос 11:
                    746: На западе США колючую проволоку часто использовали в качестве АЛЬФЫ,
                    747: чтобы не беспокоиться при возможных порывах ветра. Назовите АЛЬФУ двумя
                    748: словами.
                    749: 
                    750: Ответ:
                    751: Бельевая веревка.
                    752: 
                    753: Комментарий:
                    754: Шипы колючей проволоки легко удерживали сохнущую на ветру одежду.
                    755: 
                    756: Источник:
                    757: http://warspot.ru/2560-shipy-bez-roz
                    758: 
                    759: Автор:
                    760: Максим Евланов (Харьков)
                    761: 
                    762: Вопрос 12:
                    763: "Ты и есть мой дом" - такова, по мнению персонажа Макса Фрая, идеальная
                    764: ОНА. Премьера "ЕЕ" состоялась 30 декабря 1984 года. Назовите ЕЕ.
                    765: 
                    766: Ответ:
                    767: Формула любви.
                    768: 
                    769: Комментарий:
                    770: Во втором случае речь идет о фильме Марка Захарова.
                    771: 
                    772: Источник:
                    773:    1. М. Фрай. Сказки старого Вильнюса II.
                    774: http://www.flibusta.is/b/431642/read
                    775:    2. http://ru.wikipedia.org/wiki/Формула_любви
                    776: 
                    777: Автор:
                    778: Дмитрий Башук (Харьков)
                    779: 
                    780: Тур:
                    781: Второй игровой день. 1 тур
                    782: 
                    783: Дата:
                    784: 11-Nov-2016
                    785: 
                    786: Редактор:
                    787: Артем Матухно (Одесса)
                    788: 
                    789: Инфо:
                    790: Редактор благодарит за помощь и дельные советы во время тестирования:
                    791: Дениса Гончара, Андрея Дунаева, Ирину Буртненко, Игоря Гайдута, Сергея
                    792: Гайкова, Елену Григоращенко, Андрея Герасименко, Евгения Криворученко,
                    793: Алексея Топышева, Андрея Ландера, Влада Гаврилова.
                    794: 
                    795: Вопрос 1:
                    796: Для одной традиции компания "Гиннесс" использует вместо ЭТОГО продукцию
                    797: собственного производства. В США в годы "сухого закона" вместо ЭТОГО
                    798: использовали кока-колу. Назовите ЭТО.
                    799: 
                    800: Ответ:
                    801: Шампанское.
                    802: 
                    803: Зачет:
                    804: Бутылка шампанского.
                    805: 
                    806: Комментарий:
                    807: Речь идет о традиции "крещения корабля" с помощью бутылки шампанского.
                    808: Пивоваренная фирма "Гиннесс" использует при крещении своих судов,
                    809: предназначенных для перевозки пива, собственное пиво. В добрый путь,
                    810: друзья!
                    811: 
                    812: Источник:
                    813: http://www.rg-rb.de/index.php?option=com_rg&task=item&id=15792&Itemid=0
                    814: 
                    815: Автор:
                    816: Артем Матухно (Одесса)
                    817: 
                    818: Вопрос 2:
                    819: В 1934 году подготовленная тренером КатрИн Кёртис группа из шестидесяти
                    820: человек под названием "Современные русалки" с успехом выступила на
                    821: Всемирной выставке в Чикаго. Какое словосочетание из двух слов впервые в
                    822: истории использовал диктор НОрман Росс, комментируя это выступление?
                    823: 
                    824: Ответ:
                    825: Синхронное плавание.
                    826: 
                    827: Комментарий:
                    828: Правда, сама Катрин предпочитала термин "ритмическое плавание".
                    829: 
                    830: Источник:
                    831: David Goldblatt. How to Watch the Olympics: An Instant Initiation into
                    832: Every Sport at Rio-2016.
                    833: https://books.google.ru/books?id=RqaCCwAAQBAJ&pg=PT366#v=onepage&q&f=false
                    834: 
                    835: Автор:
                    836: Артем Матухно (Одесса)
                    837: 
                    838: Вопрос 3:
                    839: [Ведущему: начинать читать текст вопроса со слова "внимание" - это не
                    840: ошибка и не опечатка.]
                    841:    Внимание, вопрос номер два!
                    842:    Рассказывают, что некоторое время ОН, уроженец Екатеринославской
                    843: губернии, специально посещал школьные уроки, где, сидя с блокнотом на
                    844: задних партах, что-то зарисовывал. Назовите ЕГО.
                    845: 
                    846: Ответ:
                    847: [Федор Павлович] Решетников.
                    848: 
                    849: Комментарий:
                    850: Художник посещал уроки в школе перед написанием своей известной картины
                    851: "Опять двойка". Ведущий намеренно ошибся, чуть не сделав третий вопрос
                    852: "опять вторым".
                    853: 
                    854: Источник:
                    855: http://www.vokrugsveta.ru/article/252015/
                    856: 
                    857: Автор:
                    858: Артем Матухно (Одесса)
                    859: 
                    860: Вопрос 4:
                    861: Военачальника Николая Муравьёва-Карсского и декабриста Петра Колошина
                    862: объединяла настолько близкая дружба, что даже своих коней они назвали в
                    863: честь ИКСОВ. В честь ИКСОВ названы две соседние вершины-четырехтысячники
                    864: в Альпах. Назовите имена ИКСОВ.
                    865: 
                    866: Ответ:
                    867: Кастор, Поллукс.
                    868: 
                    869: Зачет:
                    870: Кастор, Полидевк.
                    871: 
                    872: Комментарий:
                    873: ИКСЫ - Диоскуры. Муравьёв-Карсский и Колошин были настолько близкими
                    874: друзьями, что часто называли друг друга братьями. Мифические герои,
                    875: близнецы Кастор и Поллукс, были известны своей неразлучной дружбой.
                    876: 
                    877: Источник:
                    878:    1. Н.Н. Муравьёв-Карсский. Собственные записки. 1811-1816.
                    879: http://www.flibusta.is/b/450099/read
                    880:    2. http://en.wikipedia.org/wiki/Castor_(mountain)
                    881:    3. http://en.wikipedia.org/wiki/Pollux_(mountain)
                    882: 
                    883: Автор:
                    884: Артем Матухно (Одесса)
                    885: 
                    886: Вопрос 5:
                    887: По словам Юрия Рылёва, автора книги "6000 изобретений XX и XXI веков,
                    888: изменивших мир", разработка противозачаточных таблеток в середине
                    889: прошлого века стала залпом "Авроры" для НЕЕ. Назовите ЕЕ точно.
                    890: 
                    891: Ответ:
                    892: Сексуальная революция.
                    893: 
                    894: Источник:
                    895: Ю.И. Рылёв. 6000 изобретений XX и XXI веков, изменившие мир.
                    896: http://www.flibusta.is/b/377039/read
                    897: 
                    898: Автор:
                    899: Артем Матухно (Одесса)
                    900: 
                    901: Вопрос 6:
                    902: Название одного магазина, предоставляющего услуги по ремонту смартфонов,
                    903: представляет собой английское слово-неологизм. Какой фрукт присутствует
                    904: на логотипе этого магазина?
                    905: 
                    906: Ответ:
                    907: Груша.
                    908: 
                    909: Зачет:
                    910: Pear.
                    911: 
                    912: Комментарий:
                    913: Магазин называется "re:Pear" [ре пЭа], что созвучно английскому глаголу
                    914: "repair" [репЭа] - чинить, ремонтировать.
                    915: 
                    916: Источник:
                    917: Личные наблюдения автора вопроса в Стокгольме.
                    918: 
                    919: Автор:
                    920: Артем Матухно (Одесса)
                    921: 
                    922: Вопрос 7:
                    923: Назовите многократного олимпийского чемпиона, чья фамилия, по одной из
                    924: версий, могла быть образована от прозвища, которым когда-то называли
                    925: слишком прилипчивого зануду.
                    926: 
                    927: Ответ:
                    928: [Евгений Викторович] Плющенко.
                    929: 
                    930: Комментарий:
                    931: Прилипчивый, словно плющ. Евгений Плющенко - многократный олимпийский
                    932: чемпион в фигурном катании.
                    933: 
                    934: Источник:
                    935:    1. Т.Ф. Ведина. Энциклопедия русских фамилий. Тайны происхождения и
                    936: значения. http://www.flibusta.is/b/154189/read
                    937:    2. http://ru.wikipedia.org/wiki/Плющенко,_Евгений_Викторович
                    938: 
                    939: Автор:
                    940: Артем Матухно (Одесса)
                    941: 
                    942: Вопрос 8:
                    943: [Ведущему: при чтении выделить интонацией словосочетание "не он".]
                    944:    Неизвестно, нравилось ли это произведение издателю, но в изначальном
                    945: авторском варианте был не ОН, а "ключ". В каком южном городе ОН
                    946: находится до сих пор?
                    947: 
                    948: Ответ:
                    949: Бахчисарай.
                    950: 
                    951: Комментарий:
                    952: Речь идет о Бахчисарайском фонтане. Пушкин изначально назвал свою поэму
                    953: "Бахчисарайский ключ", а в "фонтан" она превратилась по воле издателя -
                    954: князя Вяземского. Возможно, он считал, что авторский вариант был, как
                    955: говорится, "не фонтан".
                    956: 
                    957: Источник:
                    958: http://www.vokrugsveta.ru/vs/article/8482/
                    959: 
                    960: Автор:
                    961: Артем Матухно (Одесса)
                    962: 
                    963: Вопрос 9:
                    964: Согласно "Словарю морского жаргона", АЛЬФА - это прозвище
                    965: "привилегированной женской особы, работающей в команде судна". АЛЬФОЙ
                    966: была персонаж серии детских книг, написанных в середине прошлого века.
                    967: Назовите АЛЬФУ двумя словами.
                    968: 
                    969: Ответ:
                    970: Капитанская дочка.
                    971: 
                    972: Комментарий:
                    973: АЛЬФА - капитанская дочка. Пеппи была дочкой капитана Эфраима
                    974: Длинныйчулок.
                    975: 
                    976: Источник:
                    977:    1. Н.А. Каланов. Словарь морского жаргона. - М.: Моркнига, 2010.
                    978:    2. http://ru.wikipedia.org/wiki/Пеппи_Длинныйчулок
                    979: 
                    980: Автор:
                    981: Артем Матухно (Одесса)
                    982: 
                    983: Вопрос 10:
                    984: В одном кроссворде слово "рикша" определяется как "таксист, которого
                    985: [ДВА СЛОВА ПРОПУЩЕНО]". А кого [ДВА СЛОВА ПРОПУЩЕНО], согласно русской
                    986: пословице?
                    987: 
                    988: Ответ:
                    989: Волка.
                    990: 
                    991: Комментарий:
                    992: Пропущены слова "ноги кормят". Пословица - "Волка ноги кормят".
                    993: 
                    994: Источник:
                    995:    1. http://www.kotvet.ru/odn/145
                    996:    2. http://slovarick.ru/205/
                    997: 
                    998: Автор:
                    999: Артем Матухно (Одесса)
                   1000: 
                   1001: Вопрос 11:
                   1002: По одной из версий, ИКС - это переиначенное на английский лад выражение,
                   1003: с помощью которого французские портовые грузчики просили коллег о
                   1004: помощи. Какое слово мы заменили словом "ИКС"?
                   1005: 
                   1006: Ответ:
                   1007: Mayday.
                   1008: 
                   1009: Зачет:
                   1010: Мэйдэй; мэдэ.
                   1011: 
                   1012: Комментарий:
                   1013: Речь идет о сигнале помощи "Mayday" [мэйдэй], который представляет собой
                   1014: искаженное французское "m'aide" [мэд] - "помоги мне".
                   1015: 
                   1016: Источник:
                   1017: Ю.И. Рылёв. 6000 изобретений XX и XXI веков, изменившие мир.
                   1018: http://www.flibusta.is/b/377039/read
                   1019: 
                   1020: Автор:
                   1021: Артем Матухно (Одесса)
                   1022: 
                   1023: Вопрос 12:
                   1024: Рассказывают, что перед съемками "Терминатора" Арнольд Шварценеггер
                   1025: специально подолгу проводил время на стрельбище, дабы научиться спускать
                   1026: курок, не ДЕЛАЯ ЭТО. Как называется игра, в которой нельзя ДЕЛАТЬ ЭТО?
                   1027: 
                   1028: Ответ:
                   1029: Гляделки.
                   1030: 
                   1031: Комментарий:
                   1032: ДЕЛАТЬ ЭТО - моргать. Шварценеггер полагал, что роботы не моргают.
                   1033: 
                   1034: Источник:
                   1035:    1. https://tjournal.ru/p/terminator-30-years
                   1036:    2. http://ru.wiktionary.org/wiki/гляделки
                   1037: 
                   1038: Автор:
                   1039: Артем Матухно (Одесса)
                   1040: 
                   1041: Тур:
                   1042: Второй игровой день. 2 тур
                   1043: 
                   1044: Дата:
                   1045: 11-Nov-2016
                   1046: 
                   1047: Редактор:
                   1048: Дмитрий Петров и Михаил Локшин (Санкт-Петербург)
                   1049: 
                   1050: Инфо:
                   1051: Редакторы благодарят за тестирование вопросов и ценные замечания:
                   1052: Алексея и Марию Трефиловых, Артема Корсуна, Владимира Салия, Кристину
                   1053: Кораблину, Александра Мудрого, Сергея Терентьева, Сергея Лобачёва,
1.2       rubashki 1054: Ярослава Косарева, команду Павла Ершова и лично Павла Ершова, а также
1.1       rubashki 1055: команды "Кадис.ру", "Ноев ковчег" и "Постмодернистское название".
                   1056: 
                   1057: Вопрос 1:
                   1058: В Крыму первый ОН появился благодаря Максимилиану Волошину. По мнению
                   1059: Сергея Федина, по уму встречают только на НЕМ. Назовите ЕГО двумя
                   1060: словами.
                   1061: 
                   1062: Ответ:
                   1063: Нудистский пляж.
                   1064: 
                   1065: Комментарий:
                   1066: Там по одежке не встречают. Волошину принадлежат труды "Блики. Нагота" и
                   1067: "Блики. Маски. Нагота". И даже Ленин считал, что в движении нудистов
                   1068: есть "здоровое пролетарское начало".
                   1069: 
                   1070: Источник:
                   1071:    1. http://ru.wikipedia.org/wiki/Натуризм
                   1072:    2. http://www.gramma.ru/RST/?id=3.810
                   1073: 
                   1074: Автор:
                   1075: Михаил Локшин (Санкт-Петербург)
                   1076: 
                   1077: Вопрос 2:
                   1078: Сахарную голову можно поджечь, если с ней СДЕЛАТЬ ЭТО. Впрочем, порой
                   1079: ЭТО ДЕЛАЮТ не только с сахарной. Какие два слова на одну и ту же букву
                   1080: мы заменили словами "СДЕЛАТЬ ЭТО"?
                   1081: 
                   1082: Ответ:
                   1083: Посыпать пеплом.
                   1084: 
                   1085: Комментарий:
                   1086: Пепел (в частности, табачный) содержит соли калия, в том числе поташ,
                   1087: катализирующие горение сахарозы.
                   1088: 
                   1089: Источник:
                   1090:    1. http://www.youtube.com/watch?v=bckN7iMhjmg
                   1091:    2. http://p-i-f.livejournal.com/6982595.html
                   1092: 
                   1093: Автор:
                   1094: Михаил Локшин (Санкт-Петербург)
                   1095: 
                   1096: Вопрос 3:
                   1097: Виктор Гюго провел несколько лет в изгнании на скалистом острове Гернси.
                   1098: Бодлер насмешливо писал, что терзаемый лишь уколами тщеславия Гюго любил
                   1099: воображать себя... Кем?
                   1100: 
                   1101: Ответ:
                   1102: Прометеем.
                   1103: 
                   1104: Комментарий:
                   1105: Виктор Гюго, не принявший власть Наполеона III, был своего рода
                   1106: бунтарем, как и Прометей. А Бодлер был весьма циничен и товарищей по
                   1107: перу недолюбливал.
                   1108: 
                   1109: Источник:
                   1110: А. Труайя. Бодлер. http://www.flibusta.is/b/291671/read
                   1111: 
                   1112: Автор:
                   1113: Дмитрий Петров (Санкт-Петербург)
                   1114: 
                   1115: Вопрос 4:
                   1116: Живший в XVI веке римский папа Сикст V любил грандиозные проекты. Так,
                   1117: папа обещал оплатить ЕЕ создание, если ОНА доберется до цели. Для НЕЕ
                   1118: была разработана специальная система сигналов. Назовите ЕЕ.
                   1119: 
                   1120: Ответ:
                   1121: Непобедимая армада.
                   1122: 
                   1123: Зачет:
                   1124: Испанская армада; Великая армада.
                   1125: 
                   1126: Комментарий:
                   1127: Сикст V обещал оплатить создание армады после высадки испанцев в Англии.
                   1128: В состав флота входили корабли нескольких государств. Специально для
                   1129: Армады был создан интернациональный свод морских сигналов.
                   1130: 
                   1131: Источник:
                   1132:    1. Р. Хьюз. Рим. История города: его культура, облик, люди. - М.:
                   1133: АСТ: CORPUS, 2014. - С. 288.
                   1134:    2. http://ru.wikipedia.org/wiki/Непобедимая_армада
                   1135: 
                   1136: Автор:
                   1137: Дмитрий Петров (Санкт-Петербург)
                   1138: 
                   1139: Вопрос 5:
                   1140: Внимание, в вопросе есть замена.
                   1141:    Премьер-министр Турции на полном серьезе заявил: взгляды турецких
                   1142: мусульман отличаются от взглядов боевиков ИГИЛ на целых ДВА ИКС.
                   1143: Ответьте точно, что мы заменили словами "ДВА ИКС".
                   1144: 
                   1145: Ответ:
                   1146: 360 градусов.
                   1147: 
                   1148: Комментарий:
                   1149: Ахмет Давутоглу сказал, что взгляды турецких мусульман отличаются от
                   1150: взглядов террористов не на 180, а на целых 360 градусов. На всякий
                   1151: случай упоминаем, что террористическая организация "Исламское
                   1152: Государство" запрещена на территории РФ.
                   1153: 
                   1154: Источник:
                   1155: http://www.middleeasteye.net/news/turkish-pm-mocked-over-360-degrees-difference-slip-190994676
                   1156: 
                   1157: Автор:
                   1158: Дмитрий Петров (Санкт-Петербург)
                   1159: 
                   1160: Вопрос 6:
                   1161: Пикап Toyota Hilux [тойОта хАйлакс] надежен и прост в ремонте. По словам
                   1162: одного американского военнослужащего, Toyota Hilux стал автомобильным
                   1163: эквивалентом ЕГО. Назовите ЕГО двумя словами.
                   1164: 
                   1165: Ответ:
                   1166: Автомат Калашникова.
                   1167: 
                   1168: Комментарий:
                   1169: Toyota Hilux из-за технических характеристик чаще всего используется как
                   1170: импровизированная боевая машина в локальных конфликтах.
                   1171: 
                   1172: Источник:
                   1173:    1. http://www.businessinsider.com/why-isis-uses-toyota-trucks-2015-10
                   1174:    2. http://ru.wikipedia.org/wiki/Toyota_Hilux
                   1175: 
                   1176: Автор:
                   1177: Дмитрий Петров (Санкт-Петербург)
                   1178: 
                   1179: Вопрос 7:
                   1180: На фабрике ЕГО отца делали детали к самолетам "Мицубиси". Герой ЕГО
                   1181: произведения говорит: "Самолет - это прекрасная мечта, а конструктор -
                   1182: тот, кто ее воплощает". Назовите ЕГО.
                   1183: 
                   1184: Ответ:
                   1185: [Хаяо] Миядзаки.
                   1186: 
                   1187: Комментарий:
                   1188: Разнообразные летательные аппараты, в первую очередь самолеты, можно
                   1189: увидеть во множестве его произведений: "Небесный замок Лапута", "Ветер
                   1190: крепчает", "Навсикая из Долины Ветров", "Порко Россо" и других. Фразу из
                   1191: вопроса произносит авиаконструктор из фильма "Ветер крепчает".
                   1192: 
                   1193: Источник:
                   1194:    1. http://otium.su/xayao-miadzaki-velikij-fantazyor/
                   1195:    2. http://ru.wikipedia.org/wiki/Миядзаки,_Хаяо
                   1196:    3. Мультфильм "Ветер крепчает" (2013), реж. Хаяо Миядзаки, 12-я
                   1197: минута. http://ru.wiki2.org/wiki/Ветер_крепчает?s=The%20Wind%20Rises
                   1198: 
                   1199: Автор:
                   1200: Михаил Локшин (Санкт-Петербург)
                   1201: 
                   1202: Вопрос 8:
                   1203: Разработанный Майком Томпсоном прибор для поиска сигнала Wi-Fi выглядит
                   1204: как ОНА. Считается, что название деревни Аргури близ горы Арарат связано
                   1205: с НЕЙ. Назовите ЕЕ одним словом.
                   1206: 
                   1207: Ответ:
                   1208: Лоза.
                   1209: 
                   1210: Комментарий:
                   1211: Дизайнер решил, что поиски сигнала Wi-Fi подобны поискам воды
1.3     ! rubashki 1212: лозоходцами. Как известно, на горе Арарат зреет красный виноград.
1.1       rubashki 1213: 
                   1214: Источник:
                   1215:    1. http://www.etoday.ru/2011/10/ustroystvo-dlya-poiska-wifi--.php
                   1216:    2. http://ru.wikipedia.org/wiki/Лозоходство
                   1217:    3. http://ru.wikipedia.org/wiki/Арарат
                   1218:    4. http://ru.wikisource.org/wiki/ЭСБЕ/Аргури
                   1219: 
                   1220: Автор:
                   1221: Дмитрий Петров (Санкт-Петербург)
                   1222: 
                   1223: Вопрос 9:
                   1224: Механизмы работы иммунитета изучены недостаточно. Ученый Александр
                   1225: ХаджИдис говорит, что иммунитет - это тонкая и пока еще ТАКАЯ ОНА.
                   1226: Назовите ТАКУЮ ЕЕ.
                   1227: 
                   1228: Ответ:
                   1229: Темная материя.
                   1230: 
                   1231: Комментарий:
                   1232: Темная материя - гипотетическая форма материи, которая не испускает
                   1233: электромагнитного излучения и напрямую не взаимодействует с ним. По
                   1234: словам ученого, "иммунитет - это тонкая и пока еще темная материя".
                   1235: 
                   1236: Источник:
                   1237:    1. http://doctorpiter.ru/articles/11136/
                   1238:    2. http://ru.wikipedia.org/wiki/Тёмная_материя
                   1239: 
                   1240: Автор:
                   1241: Михаил Локшин (Санкт-Петербург)
                   1242: 
                   1243: Вопрос 10:
                   1244: Как-то раз сотрудники одного английского музея пришли на работу и
                   1245: обнаружили, что ОНА XIX века до сих пор действует. Назовите ЕЕ.
                   1246: 
                   1247: Ответ:
                   1248: Мышеловка.
                   1249: 
                   1250: Комментарий:
                   1251: Несмотря на отсутствие приманки, музейная мышь забралась в старинную
                   1252: мышеловку и не смогла оттуда выбраться.
                   1253: 
                   1254: Источник:
                   1255: http://blogs.reading.ac.uk/merl/2016/02/03/155-year-old-mouse-trap-claims-its-latest-victim/
                   1256: 
                   1257: Автор:
                   1258: Дмитрий Петров (Санкт-Петербург)
                   1259: 
                   1260: Вопрос 11:
                   1261: Единственное слово, заимствованное английским языком из швейцарского
                   1262: ретороманского, обозначает ЕЕ. Один из методов предсказания ИХ тоже
                   1263: происходит из Швейцарии. Назовите ЕЕ.
                   1264: 
                   1265: Ответ:
                   1266: Лавина.
                   1267: 
                   1268: Комментарий:
                   1269: Английское слово "avalanche" [Эвэланш] - "лавина" - пришло из
                   1270: швейцарского ретороманского через французский. Один из методов оценки
                   1271: устойчивости снежно-ледового покрова был разработан в швейцарской армии.
                   1272: 
                   1273: Источник:
                   1274:    1. Г. Доррен. Лингво. Языковой пейзаж Европы.
                   1275: http://www.flibusta.is/b/446640/read
                   1276:    2. http://ru.wikipedia.org/wiki/Лавина
                   1277: 
                   1278: Автор:
                   1279: Дмитрий Петров (Санкт-Петербург)
                   1280: 
                   1281: Вопрос 12:
                   1282: Древние китайцы верили, что ракУшки каУри приносят удачу, и придали
                   1283: ЭТОМУ форму ракУшки каУри. Если вы решите приготовить ЭТО, лучше
                   1284: использовать лазерный принтер - иначе будет трудно что-либо разобрать.
                   1285: Назовите ЭТО тремя словами.
                   1286: 
                   1287: Ответ:
                   1288: Печенье с предсказанием.
                   1289: 
                   1290: Зачет:
                   1291: Печенье с предсказаниями.
                   1292: 
                   1293: Комментарий:
                   1294: Выпекая из муки съедобное подобие каури, китайцы верили, что это
                   1295: принесет им удачу. Если использовать струйный принтер, шариковую или
                   1296: гелевую ручку, при высокой температуре текст на бумаге может смазаться.
                   1297: 
                   1298: Источник:
                   1299: http://www.orientalica.com/kitayskoe-pechene-s-predskazaniyami-retseptyi-i-istoriya
                   1300: 
                   1301: Автор:
                   1302: Михаил Локшин (Санкт-Петербург)
                   1303: 
                   1304: Тур:
                   1305: Второй игровой день. 3 тур
                   1306: 
                   1307: Дата:
                   1308: 11-Nov-2016
                   1309: 
                   1310: Редактор:
                   1311: Александр Мудрый (Черновцы)
                   1312: 
                   1313: Инфо:
                   1314: Редактор благодарит за тестирование и ценные замечания: Ростислава
                   1315: Гимчинского, Владимира Городецкого, Александра Зинченко, Ирину Зубкову,
                   1316: Андрея Кокуленко, Николая Константинова, Артема Корсуна, Александра
                   1317: Кудрявцева, Константина Науменко, Алексея и Марию Трефиловых, Игоря
                   1318: Тюнькина, Ульяну Фабричнину, а также команду "Прометей" (Черновцы).
                   1319: 
                   1320: Вопрос 1:
                   1321: Более четверти века в этом городе не функционировали полтора десятка
                   1322: станций метрополитена. Назовите этот город.
                   1323: 
                   1324: Ответ:
                   1325: Восточный Берлин.
                   1326: 
                   1327: Зачет:
                   1328: Берлин.
                   1329: 
                   1330: Комментарий:
                   1331: После сооружения Берлинской стены были закрыты семь станций на линии U6
                   1332: и восемь станций на линии U8 в связи с тем, что эти линии шли из
                   1333: западного сектора в западный через восточную часть. Было принято решение
                   1334: не разрывать линии западного метрополитена, а только закрыть станции,
1.3     ! rubashki 1335: находящиеся в восточном секторе. Поезда через эти станции двигались не
1.1       rubashki 1336: останавливаясь.
                   1337: 
                   1338: Источник:
                   1339:    1. http://ru.wikipedia.org/wiki/Берлинский_метрополитен
                   1340:    2. http://forum.tr.ru/read.php?5,174129,page=all
                   1341: 
                   1342: Автор:
                   1343: Александр Мудрый (Черновцы)
                   1344: 
                   1345: Вопрос 2:
                   1346: На мемориальной доске Траяну Поповичу, который был мэром Черновцов в
                   1347: середине прошлого века, есть число 19600. Сергей Воронцов назвал
                   1348: Поповича "буковинским ИМ". Назовите ЕГО.
                   1349: 
                   1350: Ответ:
                   1351: [Оскар] Шиндлер.
                   1352: 
                   1353: Комментарий:
                   1354: В 1941 году Попович отказался выполнить приказ о депортации черновицких
                   1355: евреев в лагеря Транснистрии. Мало того, он убедил вышестоящие власти в
                   1356: своей правоте и добился изменения решения: 19600 черновицких евреев
                   1357: остались в городе.
                   1358: 
                   1359: Источник:
                   1360:    1. http://gazeta.zn.ua/SOCIETY/bukovinskiy_shindler.html
                   1361:    2. https://pogliad.ua/news/chernivtsi/u-chernivcyah-vidkrili-memorialnu-doshku-trayanu-popovichu-280684
                   1362: 
                   1363: Автор:
                   1364: Александр Мудрый (Черновцы)
                   1365: 
                   1366: Вопрос 3:
                   1367: Большинство украинских телеканалов транслируются со спутников "Amos"
                   1368: [Эймос], "Sirius" [сИриус] и "HotBird" [хотбёд]. Поэтому самый
                   1369: распространенный на Украине тип спутниковых антенн получил сказочное
                   1370: прозвище "он". В одном из слов предыдущего предложения мы пропустили
                   1371: пять букв. Напишите это слово в исходном виде.
                   1372: 
                   1373: Ответ:
                   1374: Горыныч.
                   1375: 
                   1376: Комментарий:
                   1377: Чтобы поймать все три спутника, на тарелку устанавливают три конвертера,
                   1378: которые в просторечии называют головками. Такая "трехголовая" антенна и
                   1379: получила название в честь Змея из русских сказок.
                   1380: 
                   1381: Источник:
                   1382:    1. http://www.satsis.info/forum (поиск по слову "горыныч")
                   1383:    2. http://www.satsputnik.ru/vybrat-oborudovanie-dlya-prosmotra-besplatnogo-sputnikovogo-televideniya/
                   1384: 
                   1385: Автор:
                   1386: Александр Мудрый (Черновцы)
                   1387: 
                   1388: Вопрос 4:
                   1389: В первом тайме матча Евро-2016 Италия - Германия обе команды практически
                   1390: не предпринимали атакующих действий. Денис Казанский назвал этот тайм
                   1391: "ПРОПУСК ГарОнне". Заполните пропуск двумя или тремя словами.
                   1392: 
                   1393: Ответ:
                   1394: Стояние на реке.
                   1395: 
                   1396: Зачет:
                   1397: Стоянием на реке; стояние на; стоянием на.
                   1398: 
                   1399: Комментарий:
                   1400: Матч проходил в Бордо, расположенном на реке Гаронне, и комментатор
1.3     ! rubashki 1401: сравнил такой футбол с известными событиями русской истории.
1.1       rubashki 1402: 
                   1403: Источник:
                   1404: Трансляция четвертьфинала Евро-2016 Италия - Германия на канале "Матч!
                   1405: Футбол 1", эфир от 02.07.2016 г.
                   1406: 
                   1407: Автор:
                   1408: Александр Мудрый (Черновцы)
                   1409: 
                   1410: Вопрос 5:
                   1411: Одна из версий возникновения ЭТОГО - сбой в одном из отделов мозга. В
                   1412: такой момент новая информация поступает в подкорковые ядра, отвечающие
                   1413: за ее переработку, не напрямую, а через другую зону. Назовите ЭТО.
                   1414: 
                   1415: Ответ:
                   1416: Дежавю.
                   1417: 
                   1418: Комментарий:
                   1419: Из-за сбоя в парагиппокампальной извилине новая информация поступает в
                   1420: подкорковые ядра через правую височную долю, отвечающую за средне- и
                   1421: долговременную память. Поэтому человек воспринимает эту новую информацию
                   1422: как уже виденную.
                   1423: 
                   1424: Источник:
                   1425: http://pikabu.ru/story/prostoe_i_ponyatnoe_obyasnenie_pochemu_myi_ispyityivaem_dezhavyu_4484414
                   1426: 
                   1427: Автор:
                   1428: Ростислав Гимчинский (Черновцы)
                   1429: 
                   1430: Вопрос 6:
                   1431: После того как в болотах графства Кембриджшир было обнаружено деревянное
                   1432: колесо, эту территорию прозвали местными ИМИ. Назовите ИХ одним словом.
                   1433: 
                   1434: Ответ:
                   1435: Помпеи.
                   1436: 
                   1437: Комментарий:
                   1438: В болотах при отсутствии доступа кислорода очень хорошо сохраняются
                   1439: археологические находки. Место раскопок поселения близ Питерборо, в
                   1440: котором нашли дубовое колесо возрастом 3000 лет, сами археологи назвали
                   1441: самым богатым памятником бронзового века в Великобритании и сравнили с
                   1442: легендарными Помпеями.
                   1443: 
                   1444: Источник:
                   1445: https://www.theguardian.com/science/2016/feb/19/archaeologists-excavate-bronze-age-wheel-cambridgeshire
                   1446: 
                   1447: Автор:
                   1448: Александр Мудрый (Черновцы)
                   1449: 
                   1450: Вопрос 7:
                   1451: В одном сериале маленькая компания, разрабатывающая инновационную
                   1452: технологию, узнаёт, что у нее появился серьезный конкурент. Один из
                   1453: персонажей с грустью говорит, что их шансы опередить этого конкурента
                   1454: ЧРЕЗВЫЧАЙНО МАЛЫ. Какое соотношение мы заменили словами "ЧРЕЗВЫЧАЙНО
                   1455: МАЛЫ"?
                   1456: 
                   1457: Ответ:
                   1458: Один к гуглу.
                   1459: 
                   1460: Зачет:
                   1461: 1:10^100.
                   1462: 
                   1463: Комментарий:
                   1464: Конкурентом этой компании был "Google". Персонаж скаламбурил, понимая,
                   1465: что шансы небольшого стартапа обогнать IT-гиганта очень невелики.
                   1466: 
                   1467: Источник:
                   1468: Телесериал "Два с половиной человека", s11e16.
                   1469: 
                   1470: Автор:
                   1471: Александр Мудрый (Черновцы)
                   1472: 
                   1473: Вопрос 8:
                   1474: Внимание, ИКС в вопросе - замена.
                   1475:    Получивший образование в Париже Тадеуш Костюшко устроился ИКСОМ,
                   1476: чтобы быть поближе к любимой девушке. А заглавный персонаж известного
                   1477: произведения стал ИКСОМ, чтобы быть поближе к своему врагу. Назовите
                   1478: фамилию этого персонажа.
                   1479: 
                   1480: Ответ:
                   1481: Дубровский.
                   1482: 
                   1483: Комментарий:
                   1484: ИКС - гувернёр, учитель французского. Костюшко устроился гувернёром в
                   1485: дом богатого помещика Сосновского, в дочь которого Людвику был влюблен.
                   1486: 
                   1487: Источник:
                   1488:    1. Документальный фильм "Тадеуш Костюшко - польский генерал".
                   1489:    2. http://ru.wikipedia.org/wiki/Дубровский_(роман)
                   1490: 
                   1491: Автор:
                   1492: Александр Мудрый (Черновцы)
                   1493: 
                   1494: Вопрос 9:
                   1495: Скоростной хоккеист Виталий Абрамов в прошлом сезоне ярко ворвался в
                   1496: одну из юниорских лиг. Павел Климовицкий назвал Абрамова ИМ. Сам Абрамов
                   1497: видел ЕГО, будучи в школе. Назовите ЕГО двумя словами.
                   1498: 
                   1499: Ответ:
                   1500: Челябинский метеор.
                   1501: 
                   1502: Зачет:
                   1503: Челябинский метеорит.
                   1504: 
                   1505: Комментарий:
                   1506: Абрамов был признан лучшим новичком Главной юниорской хоккейной лиги
                   1507: Квебека (QMJHL). Поскольку Виталий из Челябинска, то его не преминули
                   1508: сравнить с метеоритом, пролетевшим над городом утром 15 февраля 2013
                   1509: года. В это время 14-летний Абрамов, как и положено, находился в школе.
                   1510: 
                   1511: Источник:
                   1512:    1. http://www.sport-express.ru/hockey/nhl/reviews/1014473/
                   1513:    2. http://www.nhl.com/ice/ru/news.htm?id=886932
                   1514: 
                   1515: Автор:
                   1516: Александр Мудрый (Черновцы)
                   1517: 
                   1518: Вопрос 10:
                   1519: Наблюдение за одной экзопланетой поставило под сомнение эффективность
                   1520: процесса циркуляризации. Заметка об этом называлась "ЧУДАКОВАТАЯ
                   1521: экзопланета задает вопросы". Какое слово мы заменили словом
                   1522: "ЧУДАКОВАТАЯ"?
                   1523: 
                   1524: Ответ:
                   1525: Эксцентричная.
                   1526: 
                   1527: Зачет:
                   1528: Эксцентрическая.
                   1529: 
                   1530: Комментарий:
                   1531: Эта планета расположена в созвездии Большой Медведицы и представляет
                   1532: собой газовый гигант размером с Юпитер. Она вращается вокруг своей
                   1533: звезды по очень вытянутой, т.е. эксцентричной орбите. Процесс
                   1534: циркуляризации состоит в "выравнивании" орбиты планеты, постепенном
                   1535: приближении ее к круговой. Но в данном случае он происходит гораздо
                   1536: медленнее, чем предсказывает теория.
                   1537: 
                   1538: Источник:
                   1539:    1. http://www.nkj.ru/news/28463/
                   1540:    2. http://dic.academic.ru/dic.nsf/dic_synonims/201591/
                   1541: 
                   1542: Автор:
                   1543: Александр Мудрый (Черновцы)
                   1544: 
                   1545: Вопрос 11:
                   1546: Рассказывая о заядлом рыбаке, Вашингтон Ирвинг пишет, что тот живет
                   1547: жизнью настоящего ЕГО. ОНИ, вероятно, происходят от кистепёрых рыб.
                   1548: Назовите ИХ двухкоренным словом.
                   1549: 
                   1550: Ответ:
                   1551: Земноводные.
                   1552: 
                   1553: Комментарий:
                   1554: Рыбак из рассказа Ирвинга в воде и около нее проводил не меньше времени,
                   1555: чем на суше. Биологи полагают, что кистепёрые рыбы дали начало
                   1556: земноводным и первыми из позвоночных вышли на сушу.
                   1557: 
                   1558: Источник:
                   1559:    1. В. Ирвинг. Происшествие с черным рыбаком.
                   1560: http://www.flibusta.is/b/70482/read
                   1561:    2. http://ru.wikipedia.org/wiki/Происхождение_земноводных
                   1562:    3. http://ru.wikipedia.org/wiki/Кистепёрые_рыбы
                   1563: 
                   1564: Автор:
                   1565: Александр Мудрый (Черновцы)
                   1566: 
                   1567: Вопрос 12:
                   1568: Назовите роман, первый перевод которого на русский язык выполнила в 1936
                   1569: году Евгения Калашникова.
                   1570: 
                   1571: Ответ:
                   1572: "Прощай, оружие!".
                   1573: 
                   1574: Комментарий:
                   1575: Вот такое любопытное совпадение. Кстати, по мнению Корнея Чуковского,
                   1576: этот перевод принадлежит к высшим достижениям советского переводческого
                   1577: искусства.
                   1578: 
                   1579: Источник:
                   1580: http://ru.wikipedia.org/wiki/Калашникова,_Евгения_Давыдовна
                   1581: 
                   1582: Автор:
                   1583: Александр Мудрый (Черновцы)
                   1584: 
                   1585: Тур:
                   1586: Третий игровой день. 1 тур
                   1587: 
                   1588: Дата:
                   1589: 09-Dec-2016
                   1590: 
                   1591: Редактор:
                   1592: Антон Волосатов и Константин Сахаров (Ивантеевка)
                   1593: 
                   1594: Инфо:
                   1595: Помощи в подготовке уделили свое ценное время: Никита Коровин (Манила),
                   1596: Дмитрий Дягилев, Татьяна Левченко, Галина Пактовская, Серафим Шибанов
                   1597: (все - Москва), Александр Усков и Олег Михеев (оба - Краснодар), Николай
                   1598: Коврижных (Киров), Екатерина Сахарова (Ивантеевка).
                   1599: 
                   1600: Вопрос 1:
                   1601: [Ведущему: отточие не озвучивать.]
                   1602:    Заметка о чемпионате по какой игре вышла под заголовком "... Без
                   1603: покемонов"?
                   1604: 
                   1605: Ответ:
                   1606: Го.
                   1607: 
                   1608: Комментарий:
                   1609: Мобильная игра "Pokémon Go" крайне популярна (или была таковой
                   1610: несколько месяцев назад), но и поклонников интеллектуальной игры го в
                   1611: мире насчитывают до 60 миллионов. Больше вопросов о покемонах и по
                   1612: заголовкам в туре не будет!
                   1613: 
                   1614: Источник:
                   1615: https://www.championat.com/other/article-252640-v-sankt-peterburge-prohodit-60-j-evropejskij-kongress-go.html
                   1616: 
                   1617: Автор:
                   1618: Константин Сахаров (Ивантеевка)
                   1619: 
                   1620: Вопрос 2:
                   1621: Однажды автор вопроса с друзьями играли в "Угадай кто", и одному из
                   1622: игроков достался популярный персонаж американского мультсериала.
                   1623: Воспроизведите первый вопрос, который задал о себе игрок.
                   1624: 
                   1625: Ответ:
                   1626: "Я в данный момент умер?".
                   1627: 
                   1628: Зачет:
                   1629: По упоминанию смерти или жизни.
                   1630: 
                   1631: Комментарий:
                   1632: Кенни из "South Park" известен тем, что постоянно умирает и воскресает,
                   1633: поэтому вопрос поставил партнеров в тупик.
                   1634: 
                   1635: Источник:
                   1636: ЛОАВ.
                   1637: 
                   1638: Автор:
                   1639: Константин Сахаров (Ивантеевка)
                   1640: 
                   1641: Вопрос 3:
                   1642: Американская чемпионка мира в детстве нередко терпела оскорбления на
                   1643: религиозной почве. Поэтому вид спорта был выбран такой, чтобы не
                   1644: испытывать проблем с экипировкой. Назовите этот вид спорта.
                   1645: 
                   1646: Ответ:
                   1647: Фехтование.
                   1648: 
                   1649: Комментарий:
                   1650: Одежда девушек-мусульман не должна открывать руки и ноги. В фехтовании
                   1651: тело и даже лицо спортсмена полностью закрыто униформой. Так что мама
                   1652: разрешила ИбтихАдж МухаммАд заниматься спортом, увидев подходящие
                   1653: костюмы фехтовальщиц. В Рио-де-Жанейро Мухаммад стала первой
                   1654: американкой, которая выступала на Олимпиаде в хиджабе.
                   1655: 
                   1656: Источник:
                   1657:    1. http://www.sports.ru/tribuna/blogs/innuendo/1020658.html
                   1658:    2. http://ru.wikipedia.org/wiki/Мухаммад,_Ибтихадж
                   1659: 
                   1660: Автор:
                   1661: Константин Сахаров (Ивантеевка)
                   1662: 
                   1663: Вопрос 4:
                   1664: Белый крест на зеленом фоне - таким предлагали сделать флаг ЕЕ. Назовите
                   1665: ЕЕ.
                   1666: 
                   1667: Ответ:
                   1668: Гренландия.
                   1669: 
                   1670: Комментарий:
                   1671: Гренландия - зависимая территория Дании, и собственный флаг ей
                   1672: предлагали сделать по аналогии со всеми скандинавскими, но с учетом
                   1673: "зеленого" названия территории. Вероятно, из-за стремления подчеркнуть
                   1674: самостоятельность нынешний флаг скандинавского креста не содержит.
                   1675: 
                   1676: Источник:
                   1677: http://ru.wikipedia.org/wiki/Флаг_Гренландии
                   1678: 
                   1679: Автор:
                   1680: Константин Сахаров (Ивантеевка)
                   1681: 
                   1682: Вопрос 5:
                   1683: Обсуждая фильм 2015 года, научный сотрудник Артем Климчук призвал ценить
                   1684: знания, потому что они могут пригодиться в самый неожиданный момент. По
                   1685: его мнению, от негативных коннотаций должно избавиться слово, означающее
                   1686: специалиста. Напишите это слово.
                   1687: 
                   1688: Ответ:
                   1689: Ботаник.
                   1690: 
                   1691: Комментарий:
                   1692: В фильме "Марсианин" главный герой - по основной профессии ботаник -
                   1693: сумел применить свои знания и выжить долгое время на Марсе. "Если
                   1694: понятие "ботаник" воспринимать не как человека, который занимается
                   1695: растениями, а как человека, который много чего знает и хорошо учился в
                   1696: институте, то краткая мораль фильма - будьте ботаниками. Это вас
                   1697: спасет".
                   1698: 
                   1699: Источник:
                   1700: http://www.the-village.ru/village/weekend/oba/223751-the-martian-mfti
                   1701: 
                   1702: Автор:
                   1703: Константин Сахаров (Ивантеевка)
                   1704: 
                   1705: Вопрос 6:
                   1706: Комментируя итоги выборов осенью 2016 года, Эльвира Набиуллина заявила:
                   1707: "Мы, конечно, все помним про песню". Из какого города группа,
                   1708: исполнившая эту песню?
                   1709: 
                   1710: Ответ:
                   1711: Из Владивостока.
                   1712: 
                   1713: Комментарий:
                   1714: Выбор городов для размещения на новых рублевых купюрах завершился в
                   1715: пользу Севастополя и Дальнего Востока. Решение о том, какой из городов
                   1716: окажется на какой купюре, будет принимать совет директоров Центробанка
                   1717: России (Набиуллина - его глава). Песня, о которой идет речь, -
                   1718: "Владивосток 2000" группы "Мумий Тролль".
                   1719: 
                   1720: Источник:
                   1721: http://www.vesti.ru/doc.html?id=2809499
                   1722: 
                   1723: Автор:
                   1724: Константин Сахаров (Ивантеевка)
                   1725: 
                   1726: Вопрос 7:
                   1727: В Эфиопии и Эритрее практикуется ОНА, включающая в себя такие стадии:
                   1728: обжарка на углях, перемалывание в деревянной ступке, варка в котле,
                   1729: перелив в другой сосуд и охлаждение, просеивание через фильтр из
                   1730: конского волоса и наконец, разлив в кипящем состоянии. Назовите ЕЕ двумя
                   1731: словами.
                   1732: 
                   1733: Ответ:
                   1734: Кофейная церемония.
                   1735: 
                   1736: Комментарий:
                   1737: Мы попытались дать описание, напоминающее более известную чайную
                   1738: церемонию. Упомянутые африканские регионы известны своим кофе.
                   1739: 
                   1740: Источник:
                   1741: http://www.intensocoffee.ru/articles.php?article=3_kofeinaja_ceremonia
                   1742: 
                   1743: Автор:
                   1744: Антон Волосатов (Ивантеевка)
                   1745: 
                   1746: Вопрос 8:
                   1747: На логотипе фирмы "Деловой звук" изображен несуществующий вид ЕГО -
                   1748: скрипичный ключ. Назовите ЕГО двумя словами.
                   1749: 
                   1750: Ответ:
                   1751: Галстучный узел.
                   1752: 
                   1753: Зачет:
                   1754: Узел галстука; способ завязывания.
                   1755: 
                   1756: Комментарий:
                   1757: Галстук - пожалуй, главнейший элемент делового стиля.
                   1758: 
                   1759: Источник:
                   1760: http://www.business-sound.ru/
                   1761: 
                   1762: Автор:
                   1763: Антон Волосатов (Ивантеевка)
                   1764: 
                   1765: Вопрос 9:
                   1766: "Самый великий человек в мире - великий вождь Ким Ир Сен". Как
                   1767: утверждает журнал "Максим", за эту фразу любого северокорейца отправили
                   1768: бы в лагерь, так как ОНО не на месте. Напишите ЕГО.
                   1769: 
                   1770: Ответ:
                   1771: Ким Ир Сен.
                   1772: 
                   1773: Зачет:
                   1774: Имя [вождя].
                   1775: 
                   1776: Комментарий:
                   1777: В КНДР имя Ким Ир Сена всегда должно стоять в начале предложения. Мы
                   1778: нарушили это правило уже дважды. :-(
                   1779: 
                   1780: Источник:
                   1781: http://www.maximonline.ru/longreads/get-smart/_article/north-korea/
                   1782: 
                   1783: Автор:
                   1784: Илья Иванов (Москва)
                   1785: 
                   1786: Вопрос 10:
                   1787: Когда ОНИ появились в Италии в начале прошлого века, за характерный
                   1788: элемент ИХ стали называть "fumetti" [фумЕтти], что означает "дымок".
                   1789: Ответьте одним словом, что такое ОНИ.
                   1790: 
                   1791: Ответ:
                   1792: Комиксы.
                   1793: 
                   1794: Комментарий:
                   1795: Итальянцам казалось, что спичбаблы в американских комиксах похожи на
                   1796: облачка дыма, так что стали именовать так и сами комиксы.
                   1797: 
                   1798: Источник:
                   1799:    1. Д.Е. Комм. Формулы страха. Введение в историю и теорию фильма
                   1800: ужасов. http://www.flibusta.is/b/353326/read
                   1801:    2. http://en.wikipedia.org/wiki/Italian_comics
                   1802: 
                   1803: Автор:
                   1804: Руслан Хаиткулов (Москва)
                   1805: 
                   1806: Вопрос 11:
                   1807: Искусствовед Эрвин ПанОфский пришел к выводу, что крылатый мальчик -
                   1808: Эрот, а его связанные руки указывают на "вынужденное целомудрие".
                   1809: Назовите заглавную героиню описанной картины.
                   1810: 
                   1811: Ответ:
                   1812: Даная.
                   1813: 
                   1814: Комментарий:
                   1815: Как известно, Зевс вступил в отношения с Данаей не какими-то привычными
                   1816: методами, а в виде золотого дождя.
                   1817: 
                   1818: Источник:
                   1819: http://ru.wikipedia.org/wiki/Даная_(картина_Рембрандта)
                   1820: 
                   1821: Автор:
                   1822: Константин Сахаров (Ивантеевка)
                   1823: 
                   1824: Вопрос 12:
                   1825: С 2016 года для НЕЕ официально введено ограничение в 45 секунд. В марте
                   1826: 1946 года Рэй МИлланд от НЕЕ отказался, ограничившись поклоном, что до
                   1827: сих пор остается уникальным случаем. Назовите ЕЕ максимально точно.
                   1828: 
                   1829: Ответ:
                   1830: Речь победителя на вручении премии "Оскар".
                   1831: 
                   1832: Зачет:
                   1833: По словам "речь" и "Оскар".
                   1834: 
                   1835: Комментарий:
                   1836: Чтобы слишком не затягивать церемонию и избежать инцидентов, выступления
                   1837: победителей регламентировали. Вручение наград традиционно проводится на
                   1838: границе зимы и весны. Несмотря на то что в карьере Милланда это была
                   1839: первая и последняя статуэтка, он проявил необычайную скромность. А
                   1840: награжден он был за лучшую мужскую роль в фильме "Потерянный уик-энд".
                   1841:    z-checkdb: Ограничение в 45 секунд действует с 2010 года, см.
                   1842: http://www.hollywoodreporter.com/lists/oscars-watch-10-longest-acceptance-867667/item/greer-garson-10-longest-acceptance-867635
                   1843: (Евгений Рубашкин).
                   1844: 
                   1845: Источник:
                   1846:    1. http://oscar2016.ru/news/45-seconds/
                   1847:    2. http://ru.wikipedia.org/wiki/Рэй_Милланд
                   1848: 
                   1849: Автор:
                   1850: Антон Волосатов (Ивантеевка)
                   1851: 
                   1852: Тур:
                   1853: Третий игровой день. 2 тур
                   1854: 
                   1855: Дата:
                   1856: 09-Dec-2016
                   1857: 
                   1858: Редактор:
                   1859: Александр Голиков и Людмила Полякова (Харьков)
                   1860: 
                   1861: Вопрос 1:
                   1862: Героиня Набокова, опустив после сложного разговора телефонную трубку,
                   1863: прислушалась и с облегчением услышала только ЕГО. Назовите ЕГО тремя
                   1864: словами, начинающимися на одну и ту же букву.
                   1865: 
                   1866: Ответ:
                   1867: Стук своего сердца.
                   1868: 
                   1869: Зачет:
                   1870: Стучание своего сердца; стук/стучание собственного сердца.
                   1871: 
                   1872: Комментарий:
                   1873: Она жила в большом доме и боялась, что этот разговор могли подслушать.
                   1874: Рядом никого не оказалось. Это не "так судьба стучится в двери", конечно
                   1875: же, но всё же.
                   1876: 
                   1877: Источник:
                   1878: В.В. Набоков. Защита Лужина. http://www.flibusta.is/b/385732/read
                   1879: 
                   1880: Автор:
                   1881: Людмила Полякова (Харьков)
                   1882: 
                   1883: Вопрос 2:
                   1884: [Ведущему: внимательно прочитать конец первого предложения, чтобы
                   1885: команды четко уяснили грамматическую структуру.]
                   1886:    В одном фильме пилигрим сообщает, что первым по доблести был Ричард
                   1887: Львиное Сердце, а ПРОПУСК. Уже в конце апреля 2016 года стало понятно,
                   1888: что, несмотря на неверие букмекеров еще осенью, ПРОПУСК - и это как
                   1889: минимум. Пропуски на слух неотличимы. Заполните любой из них двумя
                   1890: словами.
                   1891: 
                   1892: Ответ:
                   1893: Лестер второй.
                   1894: 
                   1895: Зачет:
                   1896: "Лестер" - второй.
                   1897: 
                   1898: Комментарий:
                   1899: Вторым был благородный граф Лестер.
                   1900: 
                   1901: Источник:
                   1902: Х/ф "Баллада о доблестном рыцаре Айвенго" (1982), реж. Сергей Тарасов,
                   1903: 8-я минута.
                   1904: 
                   1905: Автор:
                   1906: Александр Голиков (Харьков)
                   1907: 
                   1908: Вопрос 3:
                   1909: [Ведущему: четко прочитать "в Шуне" и "в Кване".]
                   1910:    В самом начале пьесы один из персонажей саркастично упоминает
                   1911: случайность в Шуне и случайность в Кване. Впрочем, им все-таки больше
                   1912: везет... Где?
                   1913: 
                   1914: Ответ:
                   1915: В Сычуани.
                   1916: 
                   1917: Зачет:
                   1918: В Сезуане.
                   1919: 
                   1920: Комментарий:
                   1921: Боги в произведении Бертольта Брехта уже не верят, что им удастся найти
                   1922: хотя бы одного доброго человека, но наконец-то нашелся хотя бы один - в
                   1923: городе Сычуань.
                   1924: 
                   1925: Источник:
                   1926: Б. Брехт. Добрый человек из Сычуани.
                   1927: http://www.flibusta.is/b/469976/read
                   1928: 
                   1929: Автор:
                   1930: Александр Голиков (Харьков)
                   1931: 
                   1932: Вопрос 4:
                   1933: Когда брехтовские боги обсуждают водоноса, один из них обвиняет его в
                   1934: мошенничестве, держа в руках кружку, из которой водонос их напоил водой.
                   1935: Всё дело в НЕМ. Назовите ЕГО двумя словами, начинающимися на одну и ту
                   1936: же букву.
                   1937: 
                   1938: Ответ:
                   1939: Двойное дно.
                   1940: 
                   1941: Комментарий:
                   1942: Водонос дал богам напиться воды из кружки с двойным дном, которое
                   1943: символизирует нечестность и мошенничество.
                   1944: 
                   1945: Источник:
                   1946: Б. Брехт. Добрый человек из Сычуани.
                   1947: http://www.flibusta.is/b/469976/read
                   1948: 
                   1949: Автор:
                   1950: Александр Голиков (Харьков)
                   1951: 
                   1952: Вопрос 5:
                   1953: Внимание, в одном из слов вопроса мы пропустили две буквы.
                   1954:    Вильфредо Парето - итальянский социолог-консерватор, который,
                   1955: несмотря на свой аристократизм по происхождению, пережил семейную драму.
                   1956: Преподаватель социологии, рассказывая об этом, заявил, что его жена,
                   1957: Александра Бакунина, изменила ему с маркизом. Восстановите исходное
                   1958: слово.
                   1959: 
                   1960: Ответ:
                   1961: Марксизмом.
                   1962: 
                   1963: Комментарий:
                   1964: Фамилия Александры Бакуниной явно указывает на ее марксистские и
                   1965: анархистские контакты.
                   1966: 
                   1967: Источник:
                   1968:    1. ЛНА на лекции.
                   1969:    2. http://50.economicus.ru/index.php?ch=5&le=42&r=4&z=1
                   1970:    3. http://ru.wikipedia.org/wiki/Парето,_Вильфредо
                   1971: 
                   1972: Автор:
                   1973: Александр Голиков (Харьков)
                   1974: 
                   1975: Вопрос 6:
                   1976: В мае 1949 года ОН передал свой список из 38 журналистов и писателей
                   1977: спецслужбам. В этом списке были такие характеристики, как "бесчестный
                   1978: карьерист", "сентиментальный симпатизант", "еврейский",
                   1979: "антибританский", "хорошо нажился в СССР", "очень антибелый" и тому
                   1980: подобное. Назовите ЕГО.
                   1981: 
                   1982: Ответ:
                   1983: [Эрик] Блэр.
                   1984: 
                   1985: Зачет:
                   1986: [Джордж] Оруэлл.
                   1987: 
                   1988: Комментарий:
                   1989: Да, марксист по убеждениям Эрик Блэр работал на Большого Брата.
                   1990: 
                   1991: Источник:
                   1992: http://gefter.ru/archive/15452
                   1993: 
                   1994: Автор:
                   1995: Александр Голиков (Харьков)
                   1996: 
                   1997: Вопрос 7:
                   1998: Согласно одному несерьезному источнику, в результате забастовки цыган
                   1999: тысячи горожан остались ПРОПУСК. По мнению профессора Криса Синхи,
                   2000: представители племени амондава живут ПРОПУСК, поэтому могут говорить
                   2001: только об одном событии. Какие два слова на одну и ту же букву мы дважды
                   2002: пропустили?
                   2003: 
                   2004: Ответ:
                   2005: Без будущего.
                   2006: 
                   2007: Комментарий:
                   2008: Вряд ли это племя живет в мире Оруэлла, но будущего времени в его языке
                   2009: действительно нет.
                   2010: 
                   2011: Источник:
                   2012:    1. http://www.anekdot.ru/id/256514/
                   2013:    2. http://www.bbc.com/russian/international/2011/05/110520_amondawa_language_lacks_time.shtml
                   2014: 
                   2015: Автор:
                   2016: Людмила Полякова (Харьков)
                   2017: 
                   2018: Вопрос 8:
                   2019:    <раздатка>
                   2020:    leaned
                   2021:    </раздатка>
                   2022:    Седьмой эпизод "Звездных войн" многие обвиняют во вторичности. В
                   2023: неологизме, который газета "Вашингтон Пост" применяет по отношению к
                   2024: одной из планет в этом фильме, мы убрали четыре буквы. Восстановите
                   2025: исходное слово.
                   2026: 
                   2027: Ответ:
                   2028: Alderaaned.
                   2029: 
                   2030: Комментарий:
                   2031: Планета была уничтожена, как и Алдераан, причем как способ уничтожения,
                   2032: так и кинокартинка дают основания подозревать создателей фильма во
                   2033: вторичности.
                   2034: 
                   2035: Источник:
                   2036: https://www.washingtonpost.com/news/monkey-cage/wp/2016/01/06/here-are-three-ways-that-star-wars-is-a-neoconservative-universe/
                   2037: 
                   2038: Автор:
                   2039: Людмила Полякова (Харьков)
                   2040: 
                   2041: Вопрос 9:
                   2042: Как утверждают журналисты, город Вольфсбург подхватывает воспаление
                   2043: легких, когда корпорация "Фольксваген" ДЕЛАЕТ ЭТО. Желая показать, что
                   2044: он опознал фрагменты плагиата, Джоакино Россини на концерте неоднократно
                   2045: ДЕЛАЛ ЭТО. Что делал?
                   2046: 
                   2047: Ответ:
                   2048: Снимал шляпу.
                   2049: 
                   2050: Комментарий:
                   2051: От "Фольксвагена" сильно зависит благосостояние Вольфсбурга. А Россини
                   2052: "здоровался" со "встреченными" им во время концерта коллегами.
                   2053: 
                   2054: Источник:
                   2055:    1. "Футбол", 2015, N 66 (1352). - С. 21.
                   2056:    2. А.И. Муха. Музыканты смеются. http://www.flibusta.is/b/273591/read
                   2057: 
                   2058: Автор:
                   2059: Александр Голиков (Харьков)
                   2060: 
                   2061: Вопрос 10:
                   2062: В книге "Алиса и Алисия" положительный персонаж Алиса Селезнёва
                   2063: приезжает на Суматру за одним экземпляром, путешествует во времени и
                   2064: знакомится с диктатором и тираном Алисией I. Какое слово мы пропустили в
                   2065: предыдущем предложении?
                   2066: 
                   2067: Ответ:
                   2068: Бабочки.
                   2069: 
                   2070: Комментарий:
                   2071: Как можно понять из имени, Алисия - негативное развитие Алисы.
                   2072: Получается своеобразный эффект бабочки.
                   2073: 
                   2074: Источник:
                   2075: К. Булычёв. Алиса и Алисия. http://www.flibusta.is/b/108182/read
                   2076: 
                   2077: Автор:
                   2078: Александр Голиков (Харьков)
                   2079: 
                   2080: Вопрос 11:
                   2081: Известный персонаж книги Бориса Акунина при виде Тенерифе произносит,
                   2082: как это ни странно для русскоязычного читателя, короткое слово. Назовите
                   2083: это двусложное слово.
                   2084: 
                   2085: Ответ:
                   2086: Яма.
                   2087: 
                   2088: Комментарий:
                   2089: Тенерифе - это остров со знаменитым вулканом Тейде. Персонаж, Масахиро
                   2090: Сибата, - японец, слуга Фандорина, и для него это "яма" (сравните с
                   2091: "Фудзи-яма", "Тате-яма").
                   2092: 
                   2093: Источник:
                   2094:    1. Б. Акунин. Планета Вода. http://www.flibusta.is/b/401666/read
                   2095:    2. http://ru.wikipedia.org/wiki/Тенерифе
                   2096:    3. http://ru.wikipedia.org/wiki/Фудзияма
                   2097:    4. http://ru.wikipedia.org/wiki/Татеяма_(гора)
                   2098: 
                   2099: Автор:
                   2100: Людмила Полякова (Харьков)
                   2101: 
                   2102: Вопрос 12:
                   2103: В постмодернистской пародии на пьесу "Ричард III" Эдвард расстраивается,
                   2104: что его после смерти не слушают, и упоминает о планах переезда. Ответьте
                   2105: абсолютно точно: куда именно?
                   2106: 
                   2107: Ответ:
                   2108: Эльсинор.
                   2109: 
                   2110: Зачет:
                   2111: Замок Эльсинор.
                   2112: 
                   2113: Комментарий:
                   2114: И, как и подобает в постмодернистском тексте, Эдвард становится тенью -
                   2115: видимо, тенью отца Гамлета.
                   2116: 
                   2117: Источник:
                   2118: http://the-mockturtle.livejournal.com/770332.html
                   2119: 
                   2120: Автор:
                   2121: Людмила Полякова (Харьков)
                   2122: 
                   2123: Тур:
                   2124: Третий игровой день. 3 тур
                   2125: 
                   2126: Дата:
                   2127: 09-Dec-2016
                   2128: 
                   2129: Редактор:
                   2130: Олег Михеев и Александр Усков (Краснодар)
                   2131: 
                   2132: Инфо:
                   2133: Редакторы благодарят за тестирование вопросов и ценные советы: Андрея
                   2134: Баландина и Андрея Гречишникова (оба - Москва), Максима Карачуна
                   2135: (Краснодар), Маргариту Кирюшину и Инессу Кличманову (обе - Москва), Илью
                   2136: Кукушкина и Дениса Лагутина (оба - Краснодар), Наталью Мойсик
                   2137: (Ростов-на-Дону), Марию Подрядчикову (Волгоград), Павла Солюкова
                   2138: (Краснодар), Дмитрия Тарарыкова и Артема Шинкевича (оба - Москва).
                   2139: 
                   2140: Вопрос 1:
                   2141: По легенде, художник ЗЕвксис не смог найти достаточно красивую девушку,
                   2142: для того чтобы изобразить Елену Прекрасную. Мэтью ГАмперт сравнивает
                   2143: ЗЕвксиса с заглавным героем произведения XIX века. Назовите этого героя.
                   2144: 
                   2145: Ответ:
                   2146: [Виктор] Франкенштейн.
                   2147: 
                   2148: Комментарий:
                   2149: Легенда гласит, что ЗЕвксис отобрал для позирования пятерых девушек и
                   2150: позаимствовал лучшие черты каждой из них для портрета Елены. В романе
                   2151: Мэри Шелли Франкенштейн создал своего монстра, используя различные
                   2152: фрагменты тел.
                   2153: 
                   2154: Источник:
                   2155:    1. Matthew Gumpert. Grafting Helen: The Abduction of the Classical
                   2156: Past.
                   2157: https://books.google.ru/books?id=5TM1hIAfX-EC&pg=PA258#v=onepage&q&f=false
                   2158:    2. http://ru.wikipedia.org/wiki/Виктор_Франкенштейн
                   2159: 
                   2160: Автор:
                   2161: Олег Михеев (Краснодар)
                   2162: 
                   2163: Вопрос 2:
                   2164: Желающие поучаствовать в проводимом в Сахаре марафоне должны оплатить
                   2165: вступительный взнос. В русское специализированное название того, для
                   2166: чего, помимо прочего, предназначен этот взнос, входит число. Напишите
                   2167: это число.
                   2168: 
                   2169: Ответ:
                   2170: 200.
                   2171: 
                   2172: Комментарий:
                   2173: Марафон является достаточно опасным, так что этот взнос предназначен на
                   2174: случай смерти участника и репатриации его тела на родину. В вооруженных
                   2175: силах тела погибших принято называть "грузом 200".
                   2176: 
                   2177: Источник:
                   2178:    1. https://www.si.com/vault/2002/04/29/8103066/hot--cold-at-the-marathon-des-sables-billed-as-the-worlds-toughest-footrace-even-the-stoutest-of-competitors-couldnt-avoid-getting-saharan-sand-kicked-and-blown-in-their-faces
                   2179:    2. http://ru.wikipedia.org/wiki/Груз_200
                   2180: 
                   2181: Автор:
                   2182: Олег Михеев (Краснодар)
                   2183: 
                   2184: Вопрос 3:
                   2185: Рассказывают, что каждый день с февраля 1858 года на протяжении
                   2186: четырнадцати лет Бобби можно было встретить на кладбище ГрейфрАйерс в
                   2187: Эдинбурге. В статье журнала "National Geographic" [нэшнл джиогрЭфик]
                   2188: Бобби называют предшественником... Кого?
                   2189: 
                   2190: Ответ:
                   2191: Хатико.
                   2192: 
                   2193: Комментарий:
                   2194: Скайтерьер Бобби стал знаменит тем, что охранял могилу хозяина, а затем
                   2195: был похоронен недалеко от ворот кладбища. Вскоре в Эдинбурге был открыт
                   2196: памятник Бобби. Более известна похожая история про японского пса Хатико.
                   2197: 
                   2198: Источник:
                   2199: http://www.nat-geo.ru/fact/38617-predannyy-pes-bobbi/
                   2200: 
                   2201: Автор:
                   2202: Александр Усков (Краснодар)
                   2203: 
                   2204: Вопрос 4:
                   2205: По одной из версий, альбом известной группы был назван в честь команды
                   2206: для устройств Macintosh [макинтОш]. Сейчас схожая по структуре команда
                   2207: используется в ИКСЕ. На вышеупомянутом альбоме ИКС тоже есть. Назовите
                   2208: ИКС одним словом.
                   2209: 
                   2210: Ответ:
                   2211: Android.
                   2212: 
                   2213: Зачет:
                   2214: Андроид.
                   2215: 
                   2216: Комментарий:
1.3     ! rubashki 2217: Фраза "OK Computer", ставшая названием альбома группы "Radiohead"
1.1       rubashki 2218: [рэйдиохЭд], была командой для голосового распознавания в Macintosh. В
                   2219: операционной системе Android для той же цели используется команда "OK
                   2220: Google". Композиция "Paranoid Android" [паранОид андрОид] входит в
                   2221: вышеупомянутый альбом.
                   2222: 
                   2223: Источник:
                   2224: http://ru.wikipedia.org/wiki/OK_Computer
                   2225: 
                   2226: Автор:
                   2227: Александр Усков (Краснодар)
                   2228: 
                   2229: Вопрос 5:
                   2230: По официальной версии, в Краснодаре весной этого года ОНО не состоялось
                   2231: по соображениям безопасности. Кроме того, представитель епархии упомянул
                   2232: негативные ассоциации с древним кельтским обычаем. Назовите ЕГО.
                   2233: 
                   2234: Ответ:
                   2235: Сожжение [чучела] Масленицы.
                   2236: 
                   2237: Зачет:
                   2238: Сжигание [чучела] Масленицы.
                   2239: 
                   2240: Комментарий:
                   2241: Как один из ближайших аналогов русского обычая, представители епархии
                   2242: упомянули обряд жертвоприношения друидов, в процессе которого жертва
                   2243: заключается в специальную клетку, называемую плетеным человеком, и
                   2244: подвергается сожжению.
                   2245: 
                   2246: Источник:
                   2247:    1. https://www.yuga.ru/news/392580/
                   2248:    2. http://ru.wikipedia.org/wiki/Плетёный_человек
                   2249: 
                   2250: Автор:
                   2251: Александр Усков (Краснодар)
                   2252: 
                   2253: Вопрос 6:
                   2254: В этом вопросе словом "ИКС" мы заменили другое слово.
                   2255:    Одна из статей американского закона устанавливает, в каких случаях
                   2256: кража считается совершённой в крупном размере. Например, если сумма
                   2257: украденного превышает 950 долларов или если украден ИКС. С этой статьей
                   2258: связывают происхождение фразы из трех слов, зачастую сокращаемой до
                   2259: аббревиатуры. Назовите эту аббревиатуру.
                   2260: 
                   2261: Ответ:
                   2262: GTA.
                   2263: 
                   2264: Зачет:
                   2265: ГТА.
                   2266: 
                   2267: Комментарий:
                   2268: Выражение, которым названа популярная компьютерная игра, появилось еще в
                   2269: середине века. Согласно законам Калифорнии, одним из условий, когда
                   2270: кража считается крупной, является угон автомобиля, вне зависимости от
                   2271: его цены. Так в уголовный жаргон вошло выражение "Grand Theft Auto"
                   2272: [гранд зэфт Ото]. Этим, кстати, объясняется и ненатуральный для
                   2273: английского языка порядок слов, ведь более правильным и привычным было
                   2274: бы "Auto Grand Theft".
                   2275: 
                   2276: Источник:
                   2277: http://leginfo.legislature.ca.gov/faces/codes_displaySection.xhtml?lawCode=PEN&sectionNum=487
                   2278: 
                   2279: Автор:
                   2280: Олег Михеев (Краснодар)
                   2281: 
                   2282: Вопрос 7:
                   2283: Согласно не вполне серьезному совету, британцам не стоит покупать
                   2284: красные автомобили. Дело в том, что заскучавшие полицейские иногда
                   2285: соревнуются в так называемой "дорожной" разновидности... Ответьте словом
                   2286: английского происхождения: разновидности чего?
                   2287: 
                   2288: Ответ:
                   2289: Снукер.
                   2290: 
                   2291: Комментарий:
                   2292: Снукер - это разновидность бильярда, в который играют 15 красными шарами
                   2293: и 6 шарами других цветов, причем забиваться красные и цветные шары
                   2294: должны поочередно. Популярная в Англии городская легенда гласит, что
                   2295: полицейские иногда играют в снукер машинами, т.е. останавливают машины в
                   2296: соответствии с их цветом и правилами снукера, поэтому водители красных
                   2297: автомобилей имеют наибольший шанс быть остановленными.
                   2298: 
                   2299: Источник:
                   2300:    1. http://www.exposedpolice.com/uk-traffic-police-playing-motorway-snooker/
                   2301:    2. http://ru.wikipedia.org/wiki/Снукер
                   2302: 
                   2303: Автор:
                   2304: Олег Михеев (Краснодар)
                   2305: 
                   2306: Вопрос 8:
                   2307: По одной из версий, причиной тяжелых последствий происшествия в Альпах
                   2308: могла стать установленная на НЕМ камера. На НЕМ можно было увидеть семь
                   2309: звезд. Назовите ЕГО двумя словами, начинающимися на одну и ту же букву.
                   2310: 
                   2311: Ответ:
                   2312: Шлем Шумахера.
                   2313: 
                   2314: Комментарий:
                   2315: В 2013 году Михаэль Шумахер во время катания на лыжах упал и получил
                   2316: серьезную травму головы. Существует версия, что причиной этого могла
                   2317: стать камера GoPro [гОу про], установленная на шлеме и разбившая его при
                   2318: ударе. Согласно распространенному заблуждению, семь звезд на гоночном
                   2319: шлеме Шумахера олицетворяют семь чемпионских титулов "Формулы-1", но на
                   2320: самом деле такой дизайн шлема был у него практически с начала карьеры.
                   2321: 
                   2322: Источник:
                   2323:    1. http://www.telegraph.co.uk/sport/motorsport/formulaone/michael-schumacher/10640839/Michael-Schumacher-skiing-crash-did-helmet-camera-cause-head-injuries.html
                   2324:    2. http://www.alamy.com/stock-photo-michael-schumacher-a-racing-helmet-of-the-seven-time-formula-1-world-31707308.html
                   2325: 
                   2326: Автор:
                   2327: Олег Михеев (Краснодар)
                   2328: 
                   2329: Вопрос 9:
                   2330: Согласно одному шуточному изображению, ученому удалось создать видимость
                   2331: того, что он всё еще работает над НЕЙ. Однако очевидно, что работа уже
                   2332: завершена. Назовите ЕЕ словом греческого происхождения.
                   2333: 
                   2334: Ответ:
                   2335: Голограмма.
                   2336: 
                   2337: Зачет:
                   2338: Голография.
                   2339: 
                   2340: Комментарий:
                   2341: Ученый создал голограмму самого себя, которая изображает его за работой.
                   2342: 
                   2343: Источник:
                   2344: http://www.ifunny.com/pictures/greg-was-still-hard-work-hologram-technology/
                   2345: 
                   2346: Автор:
                   2347: Александр Усков (Краснодар)
                   2348: 
                   2349: Вопрос 10:
                   2350: В начале XX века Старый Том помогал китобоям в охоте, взамен получая
                   2351: часть добычи. В одной статье это сотрудничество охарактеризовано
                   2352: термином, впервые употребленным в современном значении в 1940 году.
                   2353: Назовите этот термин словом с удвоенной согласной.
                   2354: 
                   2355: Ответ:
                   2356: Коллаборационизм.
                   2357: 
                   2358: Комментарий:
                   2359: Старый Том был вожаком стаи косаток, сопровождавших усатых китов в
                   2360: бухту, где охотились австралийские китобои. Иногда косатки даже хватали
                   2361: зубами веревку гарпуна и помогали в буксировке; взамен китобои оставляли
                   2362: им языки убитых китов. Коллаборационизм - это осознанное и добровольное
                   2363: сотрудничество с врагом. Впервые так стали называть сотрудничавшее с
                   2364: немцами правительство ВишИ во Франции.
                   2365: 
                   2366: Источник:
                   2367:    1. http://www.jewishhistoryaustralia.net/Nulla_Nulla_Story/A_5_Killers_in_Eden.htm
                   2368:    2. http://en.wikipedia.org/wiki/Collaborationism#Etymology
                   2369: 
                   2370: Автор:
                   2371: Олег Михеев (Краснодар)
                   2372: 
                   2373: Вопрос 11:
                   2374: Журналист и критик Алекс Шепард известен скептическими высказываниями.
                   2375: Одно из них привело к тому, что в октябре этого года Шепард опубликовал
                   2376: фотографию пластинки Боба Дилана, на которой лежат два металлических
                   2377: предмета. Назовите эти предметы.
                   2378: 
                   2379: Ответ:
                   2380: Вилка, нож.
                   2381: 
                   2382: Зачет:
                   2383: В любом порядке.
                   2384: 
                   2385: Комментарий:
                   2386: Боб Дилан регулярно появлялся в списке фаворитов на получение
                   2387: Нобелевской премии по литературе. Несмотря на это, Алекс Шепард
                   2388: неоднократно заявлял, что не верит в победу Дилана, а в 2015 году даже
                   2389: пообещал съесть свой экземпляр пластинки "Blood on the Tracks" [блад он
                   2390: зэ трэкс], если это всё же случится.
                   2391: 
                   2392: Источник:
                   2393:    1. https://twitter.com/alex_shephard/status/786530758571724800/
                   2394:    2. https://newrepublic.com/article/123058/who-will-win-nobel-prize-literature
                   2395: 
                   2396: Автор:
                   2397: Александр Усков (Краснодар)
                   2398: 
                   2399: Вопрос 12:
                   2400: Проведя раскопки в одном американском городе на улице КонтИ, ученые
                   2401: обнаружили, в частности, много румян и бутылок из-под ликера. Это
                   2402: позволило предположить, что заведение, упоминаемое в известном
                   2403: произведении, действительно существовало. Назовите это произведение.
                   2404: 
                   2405: Ответ:
                   2406: "The House of the Rising Sun" [чтецу: зэ хАус оф зэ рАйзин сан].
                   2407: 
                   2408: Зачет:
                   2409: Дом восходящего солнца.
                   2410: 
                   2411: Комментарий:
                   2412: "Дом восходящего солнца" - народная американская песня. Обычно под ним
                   2413: понимают то ли тюрьму, то ли бордель середины XIX века в Новом Орлеане.
                   2414: Существовало ли это заведение, до сих пор остается открытым вопросом. По
                   2415: словам археолога Шэннон Доуди, проведшей раскопки на предполагаемом
                   2416: месте, известном из газет того времени, обстановка выглядела похожей на
                   2417: бордель.
                   2418: 
                   2419: Источник:
                   2420:    1. http://en.wikipedia.org/wiki/The_House_of_the_Rising_Sun
                   2421:    2. http://www.angelpig.com/house_history.html
                   2422: 
                   2423: Автор:
                   2424: Олег Михеев (Краснодар)
                   2425: 
                   2426: Тур:
                   2427: Финал (Минск). 1 тур
                   2428: 
                   2429: Дата:
                   2430: 25-Mar-2017
                   2431: 
                   2432: Редактор:
                   2433: Ольга Кузьма и Андрей Кузьма (Санкт-Петербург)
                   2434: 
                   2435: Инфо:
                   2436: Редакторы тура благодарят за помощь и ценные замечания Алексея Рабина,
                   2437: Максима Веслополова, Евгения и Аллу Муштай (все - Санкт-Петербург) и
                   2438: Дмитрия Свинтицкого (Могилев).
                   2439: 
                   2440: Вопрос 1:
                   2441: Из первоначального списка претендентов на звание "музыкальный символ
                   2442: чемпионата мира - 2018" сразу были вычеркнуты все духовые инструменты.
                   2443: Причиной такого решения стали ОНИ. Назовите ИХ иностранным словом.
                   2444: 
                   2445: Ответ:
                   2446: Вувузелы.
                   2447: 
                   2448: Комментарий:
                   2449: Вувузелы - музыкальный символ чемпионата мира по футболу 2010 года -
                   2450: многие до сих пор вспоминают с ужасом. "Вувузела" в переводе с
                   2451: зулусского - "делать шум". Музыкальным символом ЧМ-2018 стали ложки.
                   2452: 
                   2453: Источник:
                   2454:    1. http://radiozenit.ru/news_full/uid/621
                   2455:    2. http://ru.wikipedia.org/wiki/Вувузела
                   2456: 
                   2457: Автор:
                   2458: Ольга Кузьма, Андрей Кузьма (Санкт-Петербург)
                   2459: 
                   2460: Вопрос 2:
                   2461: Уильяма Фридмана, пытавшегося предупредить ВМФ США о налёте на
                   2462: Пёрл-Харбор, называют американским ИМ. ОН родился в 1895 году в
                   2463: Бакинской губернии. Назовите ЕГО.
                   2464: 
                   2465: Ответ:
                   2466: [Рихард] Зорге.
                   2467: 
                   2468: Источник:
                   2469: Станислав Зигуненко. 100 великих загадок истории флота. - М.: Вече,
                   2470: 2012. - С. 223.
                   2471: 
                   2472: Автор:
                   2473: Ольга Кузьма, Андрей Кузьма (Санкт-Петербург)
                   2474: 
                   2475: Вопрос 3:
                   2476: Адель Алексеева пишет, что на рисунке Пушкина, в отличие от работ многих
                   2477: художников, ОН не смешной, а скорее благородный, породистый. Назовите
                   2478: ЕГО двумя словами, не используя кавычки.
                   2479: 
                   2480: Ответ:
                   2481: Нос Гоголя.
                   2482: 
                   2483: Источник:
                   2484: Адель Алексеева. Как влюблялись, творили и шалили наши классики. - М.:
                   2485: Вече, 2016. - С. 53.
                   2486: 
                   2487: Автор:
                   2488: Ольга Кузьма, Андрей Кузьма (Санкт-Петербург)
                   2489: 
                   2490: Вопрос 4:
                   2491: Известный журналист полагал, что будет похоронен рядом с человеком, чей
                   2492: девиз - "Христианство, торговля и цивилизация", но настоятель
                   2493: Вестминстерского аббатства воспротивился этому. Назовите фамилию этого
                   2494: журналиста.
                   2495: 
                   2496: Ответ:
                   2497: Стэнли.
                   2498: 
                   2499: Комментарий:
                   2500: Слово "полагал" - подсказка. Генри Мортон Стэнли нашел пропавшего в
                   2501: Африке Давида Ливингстона и приветствовал его словами "Доктор
                   2502: Ливингстон, полагаю?".
                   2503: 
                   2504: Источник:
                   2505:    1. Великие путешествия. Герои, покорившие планету. - СПб.:
                   2506: "Пресс-курьер", N 14 за 2016 год. - С. 104.
                   2507:    2. http://ru.wikipedia.org/wiki/Ливингстон,_Давид
                   2508: 
                   2509: Автор:
                   2510: Ольга Кузьма, Андрей Кузьма (Санкт-Петербург)
                   2511: 
                   2512: Вопрос 5:
                   2513: Болельщики "Манчестер Юнайтед" дали лидерам своего клуба Полю Погба,
                   2514: Златану Ибрагимовичу и Уэйну Руни очень короткое прозвище, намекающее на
                   2515: фатальность действий данных футболистов для соперников. Напишите это
                   2516: прозвище по-английски.
                   2517: 
                   2518: Ответ:
                   2519: RIP.
                   2520: 
                   2521: Комментарий:
                   2522: Прозвище образовано по первым буквам фамилий игроков. RIP - сокращение
                   2523: от "rest in peace" [рест ин пис]. "Покойся с миром" - традиционная
                   2524: надпись на надгробьях в англоязычных странах.
                   2525: 
                   2526: Источник:
                   2527: https://www.championat.com/football/news-2543880-trojke-igrokov-mju-runi-ibragimovich-pogba-dali-shutlivoe-prozvische-r-i-p.html
                   2528: 
                   2529: Автор:
                   2530: Ольга Кузьма, Андрей Кузьма (Санкт-Петербург)
                   2531: 
                   2532: Вопрос 6:
                   2533: На Каспии издавна было принято расстилать в прибрежной полосе тряпки и
                   2534: старые ковры, прижимая их камнями. Таким образом местные жители ДЕЛАЛИ
                   2535: ЭТО для домашнего использования. Россия входят в тройку лидеров среди
                   2536: стран, ДЕЛАЮЩИХ ЭТО. Какие слова мы заменили словами "ДЕЛАТЬ ЭТО"?
                   2537: 
                   2538: Ответ:
                   2539: Добывать нефть.
                   2540: 
                   2541: Источник:
                   2542: Станислав Зигуненко. 100 великих загадок истории флота. - М.: Вече,
                   2543: 2012. - С. 157.
                   2544: 
                   2545: Автор:
                   2546: Ольга Кузьма, Андрей Кузьма (Санкт-Петербург)
                   2547: 
                   2548: Вопрос 7:
                   2549: В вопросе есть замена.
                   2550:    Известный гурман Александр Дюма писал: "Я немного поохотился на
                   2551: берегах Каспия, где в таком же изобилии водятся дикие гуси, утки,
                   2552: пеликаны, как на Сене - собаки". Восстановите замененное нами слово.
                   2553: 
                   2554: Ответ:
                   2555: Лягушки.
                   2556: 
                   2557: Источник:
                   2558: Великие путешествия. Герои, покорившие планету. - СПб.: Пресс-курьер, N
                   2559: 14 за 2016 год. - С. 141.
                   2560: 
                   2561: Автор:
                   2562: Ольга Кузьма, Андрей Кузьма (Санкт-Петербург)
                   2563: 
                   2564: Вопрос 8:
                   2565: Майкл Крайтон пишет, что в период правления бельгийской колониальной
                   2566: администрации случаи каннибализма стали более редкими, а к началу 1960-х
                   2567: годов в Конго даже появилось несколько ИХ. Одно из НИХ фигурирует в
                   2568: заглавии романа 1983 года. Назовите автора этого романа.
                   2569: 
                   2570: Ответ:
                   2571: [Стивен] Кинг.
                   2572: 
                   2573: Комментарий:
                   2574: ОНИ - кладбища. Роман - "Кладбище домашних животных".
                   2575: 
                   2576: Источник:
                   2577:    1. М. Крайтон. Конго. http://flibusta.is/b/135768/read
                   2578:    2. http://ru.wikipedia.org/wiki/Кладбище_домашних_животных_(роман)
                   2579: 
                   2580: Автор:
                   2581: Ольга Кузьма, Андрей Кузьма (Санкт-Петербург)
                   2582: 
                   2583: Вопрос 9:
                   2584: Для примата галаго из семейства лориобразных слух не менее важен, чем
                   2585: зрение. За внешний вид галаго часто называют именем персонажа,
                   2586: появившегося около полувека назад. Назовите этого персонажа.
                   2587: 
                   2588: Ответ:
                   2589: Чебурашка.
                   2590: 
                   2591: Комментарий:
                   2592: У Галаго не только глаза, но и уши велики.
                   2593: 
                   2594: Источник:
                   2595: Дмитрий Бердышев. Самые необычные животные. - М.: ЭНАС-КНИГА, 2016. - С.
                   2596: 97, 101.
                   2597: 
                   2598: Автор:
                   2599: Ольга Кузьма, Андрей Кузьма (Санкт-Петербург)
                   2600: 
                   2601: Вопрос 10:
                   2602: Однажды Роберт Вуд растопил несколько килограммов свинца и вылил их в
                   2603: небольшое отверстие в земле. После того как свинец застыл, Вуд, потратив
                   2604: на раскопки несколько часов, извлек его, получив своеобразный
                   2605: скульптурный портрет ЕЕ. Назовите ЕЕ.
                   2606: 
                   2607: Ответ:
                   2608: Молния.
                   2609: 
                   2610: Комментарий:
                   2611: Разветвленная отливка ушла в землю на глубину более трех метров.
                   2612: 
                   2613: Источник:
                   2614: З. Столбовский. Великие тайны и загадки мира. Опасности и угрозы. - М.:
                   2615: Мартин, 2005. - С. 156.
                   2616: 
                   2617: Автор:
                   2618: Ольга Кузьма, Андрей Кузьма (Санкт-Петербург)
                   2619: 
                   2620: Вопрос 11:
                   2621: 26 апреля 1945 года Николай Масалов спас ребенка. Через несколько дней в
                   2622: его полк приехал человек, поговоривший с Николаем и сделавший несколько
                   2623: набросков его внешности. Назовите этого человека.
                   2624: 
                   2625: Ответ:
                   2626: [Евгений] Вучетич.
                   2627: 
                   2628: Комментарий:
                   2629: Впоследствии Вучетич создал знаменитый памятник, установленный в
                   2630: Трептов-парке.
                   2631: 
                   2632: Источник:
                   2633: Вячеслав Бондаренко. 100 великих подвигов России. - М.: Вече, 2014. - С.
                   2634: 306.
                   2635: 
                   2636: Автор:
                   2637: Ольга Кузьма, Андрей Кузьма (Санкт-Петербург)
                   2638: 
                   2639: Вопрос 12:
                   2640: В 1966 году отряд советских подводных лодок совершил кругосветное
                   2641: путешествие, не всплывая на поверхность. За время плавания советские
                   2642: моряки за ненадобностью ни разу не совершили некое действие. Шесть лет
                   2643: назад в Белоруссии это действие сочли ненужным. Назовите ЕГО.
                   2644: 
                   2645: Ответ:
                   2646: Перевод часов.
                   2647: 
                   2648: Источник:
                   2649:    1. Станислав Зигуненко. 100 великих загадок истории флота. - М.:
                   2650: Вече, 2012. - С. 246.
                   2651:    2. http://ru.wikipedia.org/wiki/Время_в_Белоруссии
                   2652: 
                   2653: Автор:
                   2654: Ольга Кузьма, Андрей Кузьма (Санкт-Петербург)
                   2655: 
                   2656: Тур:
                   2657: Финал (Минск). 2 тур
                   2658: 
                   2659: Дата:
                   2660: 25-Mar-2017
                   2661: 
                   2662: Редактор:
                   2663: Александр Кудрявцев (Николаев)
                   2664: 
                   2665: Инфо:
                   2666: Редактор благодарит за тестирование вопросов Тараса Вахрива, Дмитрия
                   2667: Великова, Анастасию Гончарову, Екатерину Дубровскую, Андрея Кокуленко,
                   2668: Юлию Лунёву, Максима Мерзлякова, Дениса Обуха, Дмитрия Овчарука, Алексея
                   2669: Рабина, Аркадия Руха и Владимира Шлапака.
                   2670: 
                   2671: Вопрос 1:
                   2672: (pic: 20160954.jpg)
                   2673:    На раздаточном материале - рентгеновский ИКС, которым чаще других
                   2674: пользовались девушки, имевшие ближневосточные или средиземноморские
                   2675: корни. Девушек привлекало то, что эффект держался всю жизнь, а процедура
                   2676: была абсолютно безболезненной. Назовите ИКС.
                   2677: 
                   2678: Ответ:
                   2679: Эпилятор.
                   2680: 
                   2681: Зачет:
                   2682: Депилятор.
                   2683: 
                   2684: Комментарий:
                   2685: У девушек с ближневосточной или средиземноморской кровью нередко на лице
                   2686: росли усики. С помощью рентгеновского облучения рост волос
                   2687: останавливался. Правда, спустя какое-то время начинали проявляться
                   2688: нежелательные последствия.
                   2689: 
                   2690: Источник:
                   2691: http://www.cosmeticsandskin.com/cdc/xray.php
                   2692: 
                   2693: Автор:
                   2694: Александр Кудрявцев (Николаев)
                   2695: 
                   2696: Вопрос 2:
                   2697: После того как французский король Филипп I обозвал Вильгельма
                   2698: Завоевателя бабой, последний некоторое время ограничивался только
                   2699: спиртными напитками, надеясь исправить ситуацию. Какое слово с удвоенной
                   2700: согласной мы пропустили в тексте вопроса?
                   2701: 
                   2702: Ответ:
                   2703: Беременной.
                   2704: 
                   2705: Комментарий:
                   2706: Вильгельм сильно растолстел и, после того как Филипп обозвал его
                   2707: беременной бабой, решил похудеть, прибегнув к диете, которая состояла
                   2708: исключительно из спиртных напитков.
                   2709: 
                   2710: Источник:
                   2711: http://www.history.com/news/history-lists/10-things-you-may-not-know-about-william-the-conqueror
                   2712: 
                   2713: Автор:
                   2714: Александр Кудрявцев (Николаев)
                   2715: 
                   2716: Вопрос 3:
                   2717: Слово "ИКС" является заменой.
                   2718:    Мать легендарного налетчика Джесси Джеймса тоже была не промах. После
                   2719: смерти сына она организовала прибыльный бизнес, скупая старые ИКСЫ и
                   2720: потом продавая их существенно дороже. Английский этимологический словарь
                   2721: сообщает, что слово "ИКС" происходит от глагола со значением "свистеть".
                   2722: Назовите ИКС.
                   2723: 
                   2724: Ответ:
                   2725: Пистолет.
                   2726: 
                   2727: Комментарий:
                   2728: Мать Джесси Джеймса обманывала покупателей, говоря, что все продаваемые
                   2729: ею пистолеты принадлежали ее сыну. Английский этимологический словарь
                   2730: указывает на происхождение слова "пистолет" от чешского
                   2731: "pi&scaron;t'ala" [пистАла], которое, в свою очередь, происходит от
                   2732: глагола "pisteti" [пистЕти] ("свистеть"). Однако если вы подумали, что
                   2733: слово происходит от английского глагола "to whistle" [ту висл], - ничего
                   2734: страшного.
                   2735: 
                   2736: Источник:
                   2737:    1. http://en.wikipedia.org/wiki/Zerelda_James
                   2738:    2. http://www.etymonline.com/index.php?term=pistol&allowed_in_frame=0
                   2739: 
                   2740: Автор:
                   2741: Александр Кудрявцев (Николаев)
                   2742: 
                   2743: Вопрос 4:
                   2744: Когда наивные телезрители стали интересоваться, как вырастить дерево у
                   2745: себя дома, компания BBC [би-би-си] ответила, что саженец следует
                   2746: посадить в жестянку с НИМ и надеяться на лучшее. Назовите ЕГО двумя
                   2747: словами, которые начинаются на соседние буквы.
                   2748: 
                   2749: Ответ:
                   2750: Томатный соус.
                   2751: 
                   2752: Комментарий:
                   2753: В 1957 году в телепрограмме BBC "Панорама" в качестве первоапрельской
                   2754: шутки был показан сюжет о необычайно богатом урожае макаронных деревьев
                   2755: в Швейцарии.
                   2756: 
                   2757: Источник:
                   2758: http://en.wikipedia.org/wiki/Spaghetti-tree_hoax
                   2759: 
                   2760: Автор:
                   2761: Александр Кудрявцев (Николаев)
                   2762: 
                   2763: Вопрос 5:
                   2764:    <раздатка>
                   2765:    низм
                   2766:    </раздатка>
                   2767:    Существует вероучение, признаЮщее и уважающее все религии, в том
                   2768: числе буддизм. Какие буквы мы пропустили в названии этого вероучения?
                   2769: 
                   2770: Ответ:
                   2771: ом.
                   2772: 
                   2773: Комментарий:
                   2774: Ом - широко употребляемая в буддизме мантра. "Omni" [Омни] - латинский
                   2775: префикс, означающий "все" или "каждый".
                   2776: 
                   2777: Источник:
                   2778:    1. http://en.wikipedia.org/wiki/Omnism
                   2779:    2. http://en.wikipedia.org/wiki/Omni
                   2780:    3. http://ru.wikipedia.org/wiki/Ом_(мантра)
                   2781: 
                   2782: Автор:
                   2783: Александр Кудрявцев (Николаев)
                   2784: 
                   2785: Вопрос 6:
                   2786: (pic: 20160955.jpg)
                   2787:    Мадрид расположен в предгорьях массива СьЕрра-де-ГвадаррАма. В статье
                   2788: об изображенной на раздаточном материале мадридской статуе сообщается и
                   2789: ее высота над уровнем моря. Укажите эту высоту.
                   2790: 
                   2791: Ответ:
                   2792: 666 метров.
                   2793: 
                   2794: Комментарий:
                   2795: Это статуя Падшего Ангела в мадридском парке БуЭн-РетИро. Согласно
                   2796: христианскому вероучению, падшие ангелы - это ангелы, взбунтовавшиеся
                   2797: против Бога и прОклятые им за это. Первым и наиболее известным падшим
                   2798: ангелом является Сатана.
                   2799: 
                   2800: Источник:
                   2801: http://en.wikipedia.org/wiki/Fuente_del_%C3%81ngel_Ca%C3%ADdo
                   2802: 
                   2803: Автор:
                   2804: Александр Кудрявцев (Николаев)
                   2805: 
                   2806: Вопрос 7:
                   2807: (pic: 20160956.jpg)
                   2808:    Через несколько часов после сооружения изображенная на снимке
                   2809: пирамида была сожжена. Назовите двумя словами причину сожжения.
                   2810: 
                   2811: Ответ:
                   2812: Сухой закон.
                   2813: 
                   2814: Зачет:
                   2815: Запрет алкоголя.
                   2816: 
                   2817: Комментарий:
                   2818: Пирамида сооружена из бочек с конфискованным алкоголем, которые через
                   2819: несколько минут будут сожжены.
                   2820: 
                   2821: Источник:
                   2822: http://www.kulturologia.ru/blogs/031115/27002/
                   2823: 
                   2824: Автор:
                   2825: Александр Кудрявцев (Николаев)
                   2826: 
                   2827: Вопрос 8:
                   2828: В прошлом веке в США группы добровольцев во избежание трагедий совершали
                   2829: поисковые рейды по свалкам и повреждали ИХ, отрывая двери или выламывая
                   2830: замки. Назовите ИХ.
                   2831: 
                   2832: Ответ:
                   2833: Холодильники.
                   2834: 
                   2835: Комментарий:
                   2836: В старых холодильниках замок не позволял открыть дверцу изнутри, и дети
                   2837: нередко погибали, забравшись внутрь во время игр. Чтобы этого случайно
                   2838: не произошло на свалке, добровольцы повреждали выброшенные холодильники.
                   2839: 
                   2840: Источник:
                   2841: http://en.wikipedia.org/wiki/Refrigerator_death
                   2842: 
                   2843: Автор:
                   2844: Александр Кудрявцев (Николаев)
                   2845: 
                   2846: Вопрос 9:
                   2847: В XVII веке английские моряки стали кое-что делать на своем корабле в
                   2848: знак скорби по погибшим товарищам, объясняя, что на время траура
                   2849: освобождают место для невидимого ИКСА смерти. Так, по одной из версий,
                   2850: появилась распространенная современная традиция. Назовите ИКС.
                   2851: 
                   2852: Ответ:
                   2853: Флаг.
                   2854: 
                   2855: Комментарий:
                   2856: Речь идет о традиции во время траура приспускать флаг.
                   2857: 
                   2858: Источник:
                   2859: https://www.washingtonpost.com/news/morning-mix/wp/2015/07/22/a-sign-of-death-not-division-the-bloody-history-behind-lowering-flags-to-half-staff/
                   2860: 
                   2861: Автор:
                   2862: Александр Кудрявцев (Николаев)
                   2863: 
                   2864: Вопрос 10:
                   2865: В 30-е годы в СССР профессия летчика считалась верхом престижа. Поэтому
                   2866: в одной телепрограмме утверждалось, что в указанное время мужчина в
                   2867: лётной форме во всех смыслах являлся ИМ. Назовите ЕГО двукоренным
                   2868: словом.
                   2869: 
                   2870: Ответ:
                   2871: Небожитель.
                   2872: 
                   2873: Источник:
                   2874: Хроники московского быта. Сын Кремля.
                   2875: http://www.tvc.ru/channel/brand/id/37/show/episodes/episode_id/45326/
                   2876: 
                   2877: Автор:
                   2878: Александр Кудрявцев (Николаев)
                   2879: 
                   2880: Вопрос 11:
                   2881: Самолет, на котором летает американский президент, называют "Air Force
                   2882: One" [эйр форс ван]. Прозвище самолета, на котором летает родившийся в
                   2883: 2012 году Американский Фараон, двумя буквами отличается от названия
                   2884: президентского самолета. Напишите это прозвище.
                   2885: 
                   2886: Ответ:
                   2887: "Air Horse One" [чтецу: эйр хорс ван].
                   2888: 
                   2889: Комментарий:
                   2890: Американский Фараон - кличка знаменитого скакуна. К местам, где проходят
                   2891: скачки, его доставляют специальным самолетом.
                   2892: 
                   2893: Источник:
                   2894:    1. http://www.eonline.com/news/663024/kentucky-derby-champion-american-pharoah-flies-on-a-plane-called-air-horse-one-yes-seriously
                   2895:    2. http://en.wikipedia.org/wiki/American_Pharoah
                   2896:    3. http://en.wikipedia.org/wiki/Air_Force_One
                   2897: 
                   2898: Автор:
                   2899: Александр Кудрявцев (Николаев)
                   2900: 
                   2901: Вопрос 12:
                   2902: (pic: 20160957.jpg)
                   2903:    Поскольку фамилией ИКСА являлось старинное английское слово,
                   2904: переводчик Владимир МЕдек использовал старочешское слово с тем же
                   2905: значением, имевшее к тому же определенную фонетическую общность.
                   2906: Назовите ИКСА.
                   2907: 
                   2908: Ответ:
                   2909: [Альбус] Дамблдор.
                   2910: 
                   2911: Комментарий:
                   2912: На раздаточном материале слово "шмель" написано на старочешском и
                   2913: современном чешском языках. Фамилия Альбуса Дамблдора происходит от
                   2914: староанглийского слова со значением "шмель"; сравните со словом
                   2915: "bumblebee" [бАмблби]. Имя "Альбус" значит "белый", поэтому шмель на
                   2916: раздаточном материале - белый.
                   2917: 
                   2918: Источник:
                   2919: http://en.wikipedia.org/wiki/Harry_Potter_in_translation
                   2920: 
                   2921: Автор:
                   2922: Александр Кудрявцев (Николаев)
                   2923: 
                   2924: Тур:
                   2925: Финал (Минск). 3 тур
                   2926: 
                   2927: Дата:
                   2928: 25-Mar-2017
                   2929: 
                   2930: Редактор:
                   2931: Алексей Полевой (Гомель) и Денис Рыбачук (Брест)
                   2932: 
                   2933: Инфо:
                   2934: Редакторы благодарят за помощь в работе над пакетом Максима Мерзлякова
                   2935: (Воронеж), Сергея Терентьева и Бориса Моносова (оба - Санкт-Петербург),
                   2936: Алексея и Марию Трефиловых (Калуга), Александра Кудрявцева (Николаев),
                   2937: Николая Слюняева (Нижний Новгород), Наиля Фарукшина (Навои - Москва),
                   2938: Дмитрия Капитанюка (Брест), Максима Новика (Гомель), Александра Огнева
                   2939: (Краков), Арсэна Атнагулова (Уфа), Тараса Вахрива (Тернополь), Игоря
                   2940: Тюнькина (Москва), Андрея Кокуленко (Омск).
                   2941: 
                   2942: Вопрос 1:
                   2943: В русском переводе одного мультфильма гигант, насмехаясь над
                   2944: противником, нараспев произносит обидное слово и повторяет его последний
                   2945: слог. Напишите это слово.
                   2946: 
                   2947: Ответ:
                   2948: Блоха.
                   2949: 
                   2950: Зачет:
                   2951: Блоха-ха-ха и т.п.
                   2952: 
                   2953: Комментарий:
                   2954: По сравнению с гигантом противник был очень мал. Переводчика этого
                   2955: эпизода, видимо, вдохновила (пауза и ... если есть возможность, то
                   2956: поставить музыкальный фрагмент отсюда (только звук, видео не надо) -
                   2957: www.youtube.com/watch?v=LXELFcq5qJE с 46 секунды) песня Мефистофеля
                   2958: композитора Мусоргского в исполнении Федора Шаляпина.
                   2959: 
                   2960: Источник:
                   2961:    1. Мультфильм "Геркулес" (студия "Дисней", 1997).
                   2962:    2. "Блоха" (Песня Мефистофеля в погребке Ауэрбаха), композитор М.П.
                   2963: Мусоргский, слова И.В. Гёте, исполняет Ф.И. Шаляпин.
                   2964: http://www.youtube.com/watch?v=LXELFcq5qJE
                   2965: 
                   2966: Автор:
                   2967: Денис Рыбачук (Брест)
                   2968: 
                   2969: Вопрос 2:
                   2970: В 1893 году иностранная фирма установила в одном из зданий российского
                   2971: города лифт. Лев КЕкушев усовершенствовал его конструкцию и получил
                   2972: премию. Что, согласно шутке современников, сделал с лифтом Кекушев?
                   2973: 
                   2974: Ответ:
                   2975: Подковал [его].
                   2976: 
                   2977: Комментарий:
                   2978: Похожая история произошла и с героем Лескова, подковавшим английскую
                   2979: блоху. Фирма, правда, была не английская, а немецкая, город - не Тула, а
                   2980: Москва, но современники шутили, что премия "за блоху, кою Лёва
                   2981: подковал".
                   2982: 
                   2983: Источник:
                   2984: http://moscow-i-ya.livejournal.com/368515.html
                   2985: 
                   2986: Автор:
                   2987: Алексей Полевой (Гомель)
                   2988: 
                   2989: Вопрос 3:
                   2990: В прошлые века при поиске залежей руды разведчики внимательно смотрели
                   2991: под ноги во время поездки. Увиденная ОНА, по мнению Тимофея БажЕнова,
                   2992: могла привести к открытию месторождения, рядом с которым запылают
                   2993: плавильные печи. Назовите ЕЕ.
                   2994: 
                   2995: Ответ:
                   2996: Искра.
                   2997: 
                   2998: Комментарий:
                   2999: Разведчики смотрели под ноги лошади. Подкованное копыто в местах,
                   3000: богатых металлическими рудами, могло высечь искру. Если месторождение
                   3001: было богатым, то вскоре из искры могло возгореться пламя плавильных
                   3002: печей. Стихотворение Пушкина с соответствующей строкой, кстати,
                   3003: называется "Во глубине сибирских руд".
                   3004: 
                   3005: Источник:
                   3006: Рейтинг Тимофея Баженова. Человек для опытов. Хозяйка медной горы.
                   3007: https://russia.tv/video/show/brand_id/3879/episode_id/170455/
                   3008: 
                   3009: Автор:
                   3010: Денис Рыбачук (Брест)
                   3011: 
                   3012: Вопрос 4:
                   3013: Герои одного романа плохо разбираются в дореволюционных реалиях. Они
                   3014: предполагают, что жившие в подвалах рабочие, чтобы выйти на солнечный
                   3015: свет, должны были купить ИКС. Какие два слова мы заменили ИКСОМ?
                   3016: 
                   3017: Ответ:
                   3018: Желтый билет.
                   3019: 
                   3020: Зачет:
                   3021: Заменительный билет.
                   3022: 
                   3023: Комментарий:
                   3024: Они думают, что билет желтый, так как по нему можно было выйти на
                   3025: солнце. На самом деле желтый билет давал возможность легально заниматься
                   3026: проституцией. Редакторы не исключают возможности того, что в зале есть
                   3027: игроки, которые знали, что официальным названием "желтого билета" было
                   3028: "заменительный билет", и ответили на вопрос, воспользовавшись
                   3029: словом-подсказкой "заменили".
                   3030: 
                   3031: Источник:
                   3032:    1. О.Д. Форш. Сумасшедший корабль. http://flibusta.is/b/268668/read
                   3033:    2. http://ru.wikipedia.org/wiki/Жёлтый_билет
                   3034: 
                   3035: Автор:
                   3036: Денис Рыбачук (Брест)
                   3037: 
                   3038: Вопрос 5:
                   3039: В биографии Ильи Ильфа отмечается, что в первые дни после переезда в
                   3040: Москву журналист искал для себя газету, отдавая предпочтение
                   3041: широкоформатным. Это объяснялось отсутствием ЕГО. Какое произведение с
                   3042: "НЕГО" начинается?
                   3043: 
                   3044: Ответ:
                   3045: Мойдодыр.
                   3046: 
                   3047: Комментарий:
                   3048: В то время Ильф был беден. Возможно, разыскивая газету пошире, чтобы
                   3049: удобнее было укрываться, он повторял строчки недавно написанной
                   3050: Чуковским сказки:
                   3051:    Одеяло убежало,
                   3052:    Улетела простыня,
                   3053:    И подушка, как лягушка,
                   3054:    Ускакала от меня.
                   3055: 
                   3056: Источник:
                   3057:    1. Александра Ильф "Дом, милый дом".
                   3058: http://tfile.co/forum/viewtopic.php?t=840279
                   3059:    2. http://www.stihi-rus.ru/1/chukovskiy/14.htm
                   3060: 
                   3061: Автор:
                   3062: Денис Рыбачук (Брест)
                   3063: 
                   3064: Вопрос 6:
                   3065: Строительство началось во время Великой Депрессии, дало работу и помогло
                   3066: выжить многим людям. Объект оказался очень дорогим - сооружение обошлось
                   3067: в 35 миллионов долларов. Назовите этот объект.
                   3068: 
                   3069: Ответ:
                   3070: Мост "Золотые ворота".
                   3071: 
                   3072: Комментарий:
                   3073: На тот момент "Золотые ворота" оказались действительно "золотыми", за
                   3074: период строительства начальная смета выросла вдвое. К сожалению, сейчас,
                   3075: находясь в депрессии, многие используют мост совсем не по назначению.
                   3076: 
                   3077: Источник:
                   3078:    1. http://gelio.livejournal.com/223154.html
                   3079:    2. http://vse-chudesa.ru/chudesa-sveta-noveyshee-vremya/most-zolotie-vorota-v-san-francisko.html
                   3080:    3. http://ru.wikipedia.org/wiki/Золотые_Ворота_(мост)
                   3081: 
                   3082: Автор:
                   3083: Алексей Полевой (Гомель)
                   3084: 
                   3085: Вопрос 7:
                   3086: В армии древних греков ИКСЫ одними из первых вступали в битву. Вскоре
                   3087: после Берлинской олимпиады копия "ИКСА" оказалась в Германии. Какое
                   3088: слово мы заменили ИКСОМ?
                   3089: 
                   3090: Ответ:
                   3091: Дискобол.
                   3092: 
                   3093: Комментарий:
                   3094: Диск использовался и как дальнее метательное оружие. Адольф Гитлер
                   3095: считал статую "Дискобола" образцом для подражания и купил в Италии одну
                   3096: из двух сохранившихся копий статуи Мирона. Кстати, статуя появляется в
                   3097: начале фильма Лени Рифеншталь "Олимпия".
                   3098: 
                   3099: Источник:
                   3100:    1. Документальный фильм "Происхождение Олимпийских игр" (Olympia -
                   3101: The Origins of Games) (2016).
                   3102: http://tfile.co/forum/viewtopic.php?t=888142
                   3103:    2. http://www.bbc.com/russian/society/2015/04/150408_vert_cul_hitlers_idea_of_the_perfect_body
                   3104: 
                   3105: Автор:
                   3106: Денис Рыбачук (Брест)
                   3107: 
                   3108: Вопрос 8:
                   3109: Сотрудник американского журнала вспоминал, как в один прекрасный день
                   3110: увидел ИКСА, который схватил АЛЬФУ быстро, как вспышка, и что обратил
                   3111: внимание на контраст между ИКСОМ и АЛЬФОЙ. Назовите ИКСА и АЛЬФУ
                   3112: словами, начинающимися на одну и ту же букву.
                   3113: 
                   3114: Ответ:
                   3115: Матрос, медсестра.
                   3116: 
                   3117: Зачет:
                   3118: Моряк, медсестра.
                   3119: 
                   3120: Комментарий:
                   3121: (pic: 20160958.jpg)
                   3122:    В один прекрасный день, когда американцы узнали об окончании войны,
                   3123: они вышли на улицы, и начались гуляния. Фотограф Альфред Эйзенштадт
                   3124: вспоминал, что матрос носился по улице быстро и целовал многих женщин,
                   3125: но кадр с медсестрой получился особенно эффектным из-за контраста между
                   3126: темной формой матроса и белым халатом медсестры. Дело было в Нью-Йорке,
                   3127: а Нью-Йорк, как известно, - город контрастов.
                   3128: 
                   3129: Источник:
                   3130: http://www.kp.by/daily/26401/3277924/
                   3131: 
                   3132: Автор:
                   3133: Алексей Полевой (Гомель)
                   3134: 
                   3135: Вопрос 9:
                   3136: Персонажа современного романа выгоняют из ТАКОГО клуба, после того как
                   3137: тот использует банку с краской в качестве импровизированной ЕЕ. Назовите
                   3138: ТАКУЮ ЕЕ.
                   3139: 
                   3140: Ответ:
                   3141: Пейнтбольная граната.
                   3142: 
                   3143: Комментарий:
                   3144: Персонаж использовал банку с краской как гранату, но в пейнтбольном
                   3145: клубе его находчивость не оценили.
                   3146: 
                   3147: Источник:
                   3148: Т. Пратчетт. Джонни и бомба. http://flibusta.is/b/486418/read
                   3149: 
                   3150: Автор:
                   3151: Денис Рыбачук (Брест)
                   3152: 
                   3153: Вопрос 10:
                   3154: КакАпо по кличке СирОкко нередко участвует в международных
                   3155: природоохранных конференциях и выставках. По мнению создателей одного
                   3156: документального фильма, СирОкко, в отличие от других какАпо, нельзя
                   3157: назвать... Каким?
                   3158: 
                   3159: Ответ:
                   3160: Нелетающим.
                   3161: 
                   3162: Комментарий:
                   3163: Новозеландские какАпо - нелетающие птицы, как и киви, но Сирокко
                   3164: является послом своего вида и символом охраны природы в Новой Зеландии,
                   3165: поэтому часто летает на различные международные конференции, правда, на
                   3166: самолетах.
                   3167: 
                   3168: Источник:
                   3169: Документальный цикл "Дикая природа Новой Зеландии" (Wild New Zealand)
                   3170: (2016), 3-я серия. http://tfile.co/forum/viewtopic.php?t=885320
                   3171: 
                   3172: Автор:
                   3173: Денис Рыбачук (Брест)
                   3174: 
                   3175: Вопрос 11:
                   3176: Яркие личинки жука мАйки, стремясь попасть в ИКС, собираются на кончиках
                   3177: травинок. Назовите ИКС.
                   3178: 
                   3179: Ответ:
                   3180: Улей.
                   3181: 
                   3182: Комментарий:
                   3183: Нелетающие, но яркие личинки образуют подобие цветка и таким образом
                   3184: приманивают пчел. Впоследствии они цепляются за пчелу и отпускают только
                   3185: тогда, когда та прилетит в улей. Последствия для пчелиной семьи бывают
                   3186: трагичными, так как личинки начинают поедать всё, что найдут в улье.
                   3187: 
                   3188: Источник:
                   3189: Документальный цикл "Яд. Достижение эволюции" (2015), 3-я серия.
                   3190: http://tfile.co/forum/viewtopic.php?t=878522
                   3191: 
                   3192: Автор:
                   3193: Денис Рыбачук (Брест)
                   3194: 
                   3195: Вопрос 12:
                   3196: В одном романе детей сначала приучают к взаимовыручке, а уже потом к
                   3197: самостоятельности. Поэтому до определенного возраста глаза у детей
                   3198: сзади, а потом - спереди. Какое слово в вопросе мы заменили?
                   3199: 
                   3200: Ответ:
                   3201: Пуговицы.
                   3202: 
                   3203: Комментарий:
                   3204: Когда все пуговицы на одежде сзади, то одеться ты можешь только с
                   3205: чьей-либо помощью, а когда спереди, то уже и самостоятельно.
                   3206: 
                   3207: Источник:
                   3208: Л. Лоури. Дающий. http://flibusta.is/b/236576/read
                   3209: 
                   3210: Автор:
                   3211: Алексей Полевой (Гомель)
                   3212: 
                   3213: Тур:
                   3214: Финал (Минск). 4 тур
                   3215: 
                   3216: Дата:
                   3217: 25-Mar-2017
                   3218: 
                   3219: Редактор:
                   3220: Серафим Шибанов (Москва), при участии Александра Карясова (Самара)
                   3221: 
                   3222: Инфо:
                   3223: Сделать пакет лучше помогали: Анастасия Белова, Глеб Крутинин, Никита
                   3224: Поздняков (все - Пущино), Виктория Бочкарёва, Сергей Кухарев, Дарья
                   3225: Макушова, Юрий Мотькин, Регина Шарипова (все - Самара), Александр
                   3226: Кудрявцев (Николаев), а также команды "Приматы" и "Слон потрогает тебя"
                   3227: (обе - Самара).
                   3228: 
                   3229: Вопрос 1:
                   3230: Компания "Alphabet" [Альфабет] перенесла свой сайт на новый домен,
                   3231: который позиционируется как альтернатива .com [дот ком]. По мнению
                   3232: автора вопроса, схожим образом в своем логотипе подчеркивает
                   3233: разнообразие предоставляемых услуг и другая компания. Назовите эту
                   3234: другую компанию.
                   3235: 
                   3236: Ответ:
                   3237: "Amazon".
                   3238: 
                   3239: Комментарий:
                   3240: Новый сайт "Alphabet" [Альфабет] находится по адресу abc.xyz [эй-би-си
                   3241: дот икс-вай-зет], т.е., можно сказать, от A [эй] до Z [зет]. В логотипе
                   3242: "AmaZon" [амазОн] от буквы A [эй] к букве Z [зет] идет стрелочка.
                   3243: 
                   3244: Источник:
                   3245:    1. https://abc.xyz/
                   3246:    2. http://en.wikipedia.org/wiki/Alphabet_Inc.
                   3247:    3. https://www.amazon.co.uk/
                   3248: 
                   3249: Автор:
                   3250: Илья Иванов (Путилково)
                   3251: 
                   3252: Вопрос 2:
                   3253: "Гамбург" - единственный футбольный клуб, который играл во всех сезонах
                   3254: немецкой Бундеслиги. За это клуб получил прозвище "ОН". Назовите фильм
                   3255: 1993 года, в котором можно увидеть более десятка ИХ.
                   3256: 
                   3257: Ответ:
                   3258: "Парк Юрского периода".
                   3259: 
                   3260: Комментарий:
                   3261: Клуб прозвали Динозавром (иногда встречается вариант "Динозавр
                   3262: Бундеслиги"). В фильме "Парк Юрского периода" много разных динозавров.
                   3263: 
                   3264: Источник:
                   3265:    1. http://en.wikipedia.org/wiki/Hamburger_SV
                   3266:    2. http://en.wikipedia.org/wiki/Jurassic_Park_(film)
                   3267: 
                   3268: Автор:
                   3269: Серафим Шибанов (Москва)
                   3270: 
                   3271: Вопрос 3:
                   3272: В песне группы "25/17" [двадцать пять семнадцать] отмечается, что
                   3273: реальность и так достаточно сурова, и говорится, что бояться нужно не
                   3274: ВТОРЫХ, а ПЕРВЫХ. Кто создал произведение, в заглавии которого
                   3275: фигурируют ПЕРВЫЕ и ВТОРЫЕ?
                   3276: 
                   3277: Ответ:
                   3278: [Константин] Симонов.
                   3279: 
                   3280: Комментарий:
                   3281: Строчка из песни звучит так: "Все боятся мертвых, а нужно живых".
                   3282: 
                   3283: Источник:
                   3284:    1. http://rusrap.org.ru/mp3/2517/text/Cherep_i_kosti.htm
                   3285:    2. http://ru.wikipedia.org/wiki/Живые_и_мёртвые_(роман)
                   3286: 
                   3287: Автор:
                   3288: Серафим Шибанов (Москва)
                   3289: 
                   3290: Вопрос 4:
                   3291: Надеемся, этот вопрос будет вам приятен.
                   3292:    В конце XIX века Лондон часто называли выражением, которое буквально
                   3293: переводится как "Большой ОН". Согласно поговорке, ОН тесно связан... С
                   3294: чем?
                   3295: 
                   3296: Ответ:
                   3297: С огнем.
                   3298: 
                   3299: Комментарий:
                   3300: Лондон известен своим смогом, поэтому получил прозвище Big Smoke [биг
                   3301: смОук], что буквально переводится как "Большой дым". Согласно поговорке,
                   3302: дыма без огня не бывает. В начале вопроса есть отсылка к цитате из "Горя
                   3303: от ума" "И дым отечества нам сладок и приятен".
                   3304: 
                   3305: Источник:
                   3306:    1. Дж.Э. Гарднер. Возвращение Мориарти.
                   3307: http://flibusta.is/b/264025/read
                   3308:    2. http://slovarick.ru/374/
                   3309: 
                   3310: Автор:
                   3311: Серафим Шибанов (Москва)
                   3312: 
                   3313: Вопрос 5:
                   3314: В своей книге Крис Тёрни поступил с погубившим древнюю сосну ученым так,
                   3315: как должны были поступить с НИМ. В каком городе ОН жил?
                   3316: 
                   3317: Ответ:
                   3318: Эфес.
                   3319: 
                   3320: Комментарий:
                   3321: Крис Тёрни решил, что не стоит упоминать имя ученого, уничтожившего
                   3322: столь ценный объект. Тот, кого древние греки запретили вспоминать, жил в
                   3323: Эфесе и сжег местный храм Артемиды.
                   3324: 
                   3325: Источник:
                   3326:    1. К. Тёрни. Кости, скалы и звезды. Наука о том, когда что произошло.
                   3327: http://flibusta.is/b/320384/read
                   3328:    2. http://en.wikipedia.org/wiki/Herostratus
                   3329: 
                   3330: Автор:
                   3331: Игорь Тюнькин (Москва)
                   3332: 
                   3333: Вопрос 6:
                   3334: Футболист МАриан ЧИшовски окончил карьеру в 2014 году. Незадолго до
                   3335: завершения карьеры спортсмен принял участие в ЭТОМ с подачи хоккеиста
                   3336: ВАцлава ПлЕтки. Назовите ЭТО тремя английскими словами.
                   3337: 
                   3338: Ответ:
                   3339: Ice Bucket Challenge [чтецу: айс бАкет чЕллендж].
                   3340: 
                   3341: Зачет:
                   3342: Айс бакет челлендж.
                   3343: 
                   3344: Комментарий:
                   3345: Чишовски принял участие в флешмобе под названием "Ice Bucket Challenge"
                   3346: [айс бакет чЕллендж], который должен был привлечь внимание к боковому
                   3347: амиотрофическому склерозу. К сожалению, через пару месяцев после этого
                   3348: Мариан узнал, что сам болен этой болезнью.
                   3349: 
                   3350: Источник:
                   3351: http://www.sports.ru/tribuna/blogs/superratings/1182036.html
                   3352: 
                   3353: Автор:
                   3354: Александр Карясов (Самара)
                   3355: 
                   3356: Вопрос 7:
                   3357: [Ведущему: выделить голосом слово "распалась".]
                   3358:    В 1918 году ЕМУ предложили должность в университете города Черновцы.
                   3359: Пока ОН думал, ехать или не ехать, Австро-Венгрия распалась, и ОН
                   3360: отказался. Назовите ЕГО.
                   3361: 
                   3362: Ответ:
                   3363: [Эрвин] Шрёдингер.
                   3364: 
                   3365: Комментарий:
                   3366: А если бы Австро-Венгрия не распалась, Шрёдингер принял бы это
                   3367: предложение. В знаменитом мысленном эксперименте кот жив, если атом
                   3368: радиоактивного вещества не распался, и мертв, если распался.
                   3369: 
                   3370: Источник:
                   3371: http://ru.wikipedia.org/wiki/Шрёдингер,_Эрвин
                   3372: 
                   3373: Автор:
                   3374: Александр Карясов (Самара)
                   3375: 
                   3376: Вопрос 8:
                   3377:    <раздатка>
                   3378:    В памятник Захер-Мазоху во Львове вмонтирована ОНА, через которую
                   3379: можно разглядывать сменяющиеся эротические картинки. ONA изображена на
                   3380: памятнике в европейской столице вместе... С кем?
                   3381:    </раздатка>
                   3382:    Вопрос перед вами. Время!
                   3383: 
                   3384: Ответ:
                   3385: С Ромулом и Ремом.
                   3386: 
                   3387: Комментарий:
                   3388: В первом случае имеется в виду лупа как прибор для улучшения зрения, а
                   3389: во втором - Lupa capitolina - "Капитолийская волчица".
                   3390: 
                   3391: Источник:
                   3392:    1. http://www.travel.ru/news/2008/03/24/121343.html
                   3393:    2. http://it.wikipedia.org/wiki/Lupa_capitolina
                   3394: 
                   3395: Автор:
                   3396: Александр Карясов (Самара)
                   3397: 
                   3398: Вопрос 9:
                   3399: Советский ученый Владимир СперАнтов рассказывал, как его смелый знакомый
                   3400: заявил, что устал от происходящего, и ушел к Сергею. Назовите фамилию
                   3401: Сергея.
                   3402: 
                   3403: Ответ:
                   3404: Прокофьев.
                   3405: 
                   3406: Комментарий:
                   3407: Сергей Генкин, как и многие советские люди, присутствовал на похоронах
                   3408: Сталина. Однако ему надоела толпа людей, и поэтому он захотел пойти на
                   3409: похороны Прокофьева, который также умер 5 марта 1953 года.
                   3410: 
                   3411: Источник:
                   3412: http://050353.ru/2016/03/04/sperantov/
                   3413: 
                   3414: Автор:
                   3415: Александр Карясов, в редакции Юрия Мотькина (оба - Самара)
                   3416: 
                   3417: Вопрос 10:
                   3418: ЕЕ дочерей зовут ОктАвия и СеверИна. В одном интервью ОНА призналась,
                   3419: что в детстве была на домашнем обучении. Кто ОНА?
                   3420: 
                   3421: Ответ:
                   3422: [Валерия Гай] Германика.
                   3423: 
                   3424: Комментарий:
                   3425: Создательница сериала "Школа" сама в школу толком и не ходила.
                   3426: 
                   3427: Источник:
                   3428:    1. http://ru.wikipedia.org/wiki/Германика,_Валерия_Гай_Александровна
                   3429:    2. http://www.razgovorchiki.ru/arkhiv/germanika.htm
                   3430: 
                   3431: Автор:
                   3432: Серафим Шибанов (Москва)
                   3433: 
                   3434: Вопрос 11:
                   3435: В статье о НЕМ упоминается старовенгерский алфавит, придуманный
                   3436: пастухами. Напишите слово, которое мы обозначили как ОН, при помощи
                   3437: НЕГО.
                   3438: 
                   3439: Ответ:
                   3440:    Бустро
                   3441:    нодеф
                   3442: 
                   3443: Зачет:
                   3444: Часть слова "бустрофедон" должна быть написана в одном направлении,
                   3445: часть - в другом.
                   3446: 
                   3447: Комментарий:
                   3448: Бустрофедон - способ письма, при котором чередуются строки, идущие слева
                   3449: направо и справа налево.
                   3450: 
                   3451: Источник:
                   3452: http://en.wikipedia.org/wiki/Boustrophedon
                   3453: 
                   3454: Автор:
                   3455: Александр Карясов (Самара)
                   3456: 
                   3457: Вопрос 12:
                   3458: Сериал "Во все тяжкие" повествует об Уолтере Уайте, которому
                   3459: диагностируют неоперабельный рак. Вскоре после окончания сериала в
                   3460: издании "Albuquerque Journal" [альбукЕрке джОрнал] появился ОН. Назовите
                   3461: ЕГО точно.
                   3462: 
                   3463: Ответ:
                   3464: Некролог [Уолтера] Уайта.
                   3465: 
                   3466: Комментарий:
                   3467: Фанаты не поленились, и не важно, что Уолтер Уайт был всего лишь
                   3468: персонаж.
                   3469: 
                   3470: Источник:
                   3471: http://variety.com/2013/tv/news/breaking-bad-walter-white-obit-1200694265/
                   3472: 
                   3473: Автор:
                   3474: Серафим Шибанов (Москва)
                   3475: 
                   3476: Тур:
                   3477: Финал (Минск). 5 тур
                   3478: 
                   3479: Дата:
                   3480: 25-Mar-2017
                   3481: 
                   3482: Редактор:
                   3483: Мишель Матвеев (Санкт-Петербург)
                   3484: 
                   3485: Инфо:
                   3486: Редактор благодарит тестеров: Антон Тахтаров, Яна Азриэль, Дмитрий
                   3487: Овчарук, Наталья Орлова, Сергей Терентьев, Александр Коробейников,
                   3488: Владимир Бройда, Александр Камаев, Юлия Фукельман, Сергей Лобачёв,
                   3489: Владимир Городецкий, Ирина Зубкова, Антон Волосатов, Валерий Юдачёв,
                   3490: Илья Чадаев, Андрей Данченко, Алексей Акименко, Глеб Олейник, Андрей
                   3491: Кокуленко, Ваган Калайджян, Садиг Гамидов, Артем Рожков.
                   3492: 
                   3493: Вопрос 1:
                   3494: Один человек был изуродован своими врагами, но в Петербурге нашелся
                   3495: искусный специалист, который ему помог. Исследователь Крашенинников
                   3496: усматривает в этом основу сюжета произведения. Какого?
                   3497: 
                   3498: Ответ:
                   3499: "Нос".
                   3500: 
                   3501: Комментарий:
                   3502: Упомянутому человеку враги отрезали нос, а петербургский мастер
                   3503: изготовил для него протез. По мнению Крашенинникова, искусственный нос
                   3504: мог навеять Гоголю фантазию о самостоятельных прогулках носа коллежского
                   3505: асессора Ковалева.
                   3506: 
                   3507: Источник:
                   3508: http://magazines.russ.ru/voplit/2001/5/kpas.html
                   3509: 
                   3510: Автор:
                   3511: Мишель Матвеев (Санкт-Петербург)
                   3512: 
                   3513: Вопрос 2:
                   3514: ОНИ сильно ограничивают свободу действий и поэтому, согласно Википедии,
                   3515: стали символом женского целомудрия и мужской власти. Назовите процедуру,
                   3516: результатом которой ОНИ становились.
                   3517: 
                   3518: Ответ:
                   3519: Бинтование ног.
                   3520: 
                   3521: Зачет:
                   3522: Забинтовывание ног; перебинтовывание ног; бинтовка ног.
                   3523: 
                   3524: Комментарий:
                   3525: Женщина с ногами, изуродованными бинтованием, была ограничена в
                   3526: возможности передвигаться самостоятельно, она была вынуждена сидеть дома
                   3527: и не могла пойти куда-либо без сопровождения слуг.
                   3528: 
                   3529: Источник:
                   3530: http://ru.wikipedia.org/wiki/Бинтование_ног
                   3531: 
                   3532: Автор:
                   3533: Мишель Матвеев (Санкт-Петербург)
                   3534: 
                   3535: Вопрос 3:
                   3536: Право посетить непосредственно сам ИКС получает очень небольшое
                   3537: количество людей. В частности, один раз это могут сделать нобелевские
                   3538: лауреаты. Назовите ИКС двумя словами.
                   3539: 
                   3540: Ответ:
                   3541: Корабль "Ваза".
                   3542: 
                   3543: Зачет:
                   3544: Корабль "Васа".
                   3545: 
                   3546: Комментарий:
                   3547: Музей корабля "Ваза" в Стокгольме ежедневно посещает множество людей, но
                   3548: экспозиция расположена вокруг корабля, а сам корабль берегут и
                   3549: посетителей на него пускают лишь в исключительных случаях. В частности,
                   3550: раз в году после вручения Нобелевской премии лауреаты получают право на
                   3551: посещение.
                   3552: 
                   3553: Источник:
                   3554:    1. Рассказ экскурсовода в музее "Ваза".
                   3555:    2. http://cr2.livejournal.com/84353.html
                   3556: 
                   3557: Автор:
                   3558: Мишель Матвеев (Санкт-Петербург)
                   3559: 
                   3560: Вопрос 4:
                   3561: К герою Акунина, лежащему в реанимации после автокатастрофы, приходит
                   3562: посетитель. Белье героя он называет словом женского рода. Каким?
                   3563: 
                   3564: Ответ:
                   3565: Рубашка.
                   3566: 
                   3567: Зачет:
                   3568: Сорочка.
                   3569: 
                   3570: Комментарий:
                   3571: Идиома "родиться в рубашке" означает "быть везучим, счастливо избегать
                   3572: беды". По мнению посетителя, белье героя, выжившего в автокатастрофе, -
                   3573: это и есть та рубашка, в которой он родился.
                   3574: 
                   3575: Источник:
                   3576: Б. Акунин. Фантастика. http://flibusta.is/b/182057/read
                   3577: 
                   3578: Автор:
                   3579: Мишель Матвеев (Санкт-Петербург)
                   3580: 
                   3581: Вопрос 5:
                   3582: На знаменитом мероприятии охрана может проверить любого посетителя,
                   3583: выходящего из палатки. Так предотвращаются тысячи ИХ. Назовите ИХ тремя
                   3584: словами.
                   3585: 
                   3586: Ответ:
                   3587: Кражи пивных кружек.
                   3588: 
                   3589: Зачет:
                   3590: Похищения пивных кружек; кражи/похищения пивных бокалов.
                   3591: 
                   3592: Комментарий:
                   3593: Мероприятие - Октоберфест. Кружки крадут не только ради кружек, но еще и
                   3594: просто потому, что это стало популярным развлечением. Приходится
                   3595: принимать меры.
                   3596: 
                   3597: Источник:
                   3598: http://ru.wikipedia.org/wiki/Октоберфест
                   3599: 
                   3600: Автор:
                   3601: Мишель Матвеев (Санкт-Петербург)
                   3602: 
                   3603: Вопрос 6:
                   3604: Микробиолог Блэкмор выяснил, что некоторые бактерии могут двигаться
                   3605: необычным образом благодаря содержащимся в их организме кристаллическим
                   3606: цепочкам железа. Аналогом чего являются эти цепочки?
                   3607: 
                   3608: Ответ:
                   3609: Компаса.
                   3610: 
                   3611: Комментарий:
                   3612: Эти бактерии могут двигаться строго на север или на юг, поскольку
                   3613: упомянутые цепочки вытягиваются вдоль линий магнитного поля Земли.
                   3614: 
                   3615: Источник:
                   3616:    1. С.И. Венецкий. В мире металлов. http://flibusta.is/b/224963/read
                   3617:    2. http://ru.wikipedia.org/wiki/Магнетосома
                   3618: 
                   3619: Автор:
                   3620: Мишель Матвеев (Санкт-Петербург)
                   3621: 
                   3622: Вопрос 7:
                   3623: Сергей Бубка пошутил, что, когда в одном государстве происходит некое
                   3624: событие, стоит такой рев, что его слышно в соседних государствах.
                   3625: Назовите это событие.
                   3626: 
                   3627: Ответ:
                   3628: Гран-при Монако.
                   3629: 
                   3630: Зачет:
                   3631: Гонки "Формула-1" и т.п. по словам "Формула-1".
                   3632: 
                   3633: Комментарий:
                   3634: Имеется в виду рев моторов. Монако - совсем маленький анклав внутри
                   3635: Франции, а расстояние до итальянской границы составляет всего десяток
                   3636: километров. Сергей Бубка, как и ряд других спортсменов, по окончании
                   3637: карьеры поселился в Монако.
                   3638: 
                   3639: Источник:
                   3640: https://esquire.ru/wil/bubka
                   3641: 
                   3642: Автор:
                   3643: Мишель Матвеев (Санкт-Петербург)
                   3644: 
                   3645: Вопрос 8:
                   3646: Дуплет.
                   3647:    1. Персонаж одного романа называл ЕЕ Венерой. Назовите ЕЕ двумя
                   3648: словами.
                   3649:    2. Персонажу классического произведения в критический момент
                   3650: показалось, что ОНА - это аристократка. Назовите ЕЕ двумя словами.
                   3651: 
                   3652: Ответ:
                   3653:    1. Дама червей.
                   3654:    2. Дама пик.
                   3655: 
                   3656: Комментарий:
                   3657:    1. Венера - богиня любви, сердце - символ любви и символ червовой
                   3658: масти.
                   3659:    2. В "Пиковой даме" Германну в момент проигрыша померещилось, что
                   3660: дама пик была старухой-графиней.
                   3661: 
                   3662: Источник:
                   3663:    1. Б. Акунин. Азазель. http://flibusta.is/b/228606/read
                   3664:    2. А.С. Пушкин. Пиковая дама.
                   3665: http://www.rvb.ru/pushkin/01text/06prose/01prose/0866.htm
                   3666: 
                   3667: Автор:
                   3668: Мишель Матвеев (Санкт-Петербург)
                   3669: 
                   3670: Вопрос 9:
                   3671: Гравитон - это гипотетическая частица, переносящая гравитационное
                   3672: взаимодействие. Часто задаются вопросом, почему ОН не оказывается
                   3673: непреодолимым для гравитонов. Назовите ЕГО.
                   3674: 
                   3675: Ответ:
                   3676: Горизонт событий [черной дыры].
                   3677: 
                   3678: Комментарий:
                   3679: С одной стороны, черная дыра гравитационно взаимодействует с окружающим
                   3680: миром. С другой стороны, согласно общей теории относительности, ничто, в
                   3681: числе гравитон, не может выбраться из-за ее горизонта событий. Есть
                   3682: несколько возможных способов разрешения этого противоречия - например,
                   3683: гравитация может распространяться виртуальными гравитонами, рождающимися
                   3684: вблизи горизонта событий. Окончательный ответ, видимо, зависит от того,
                   3685: какой будет теория, в рамках которой будет описан гравитон.
                   3686: 
                   3687: Источник:
                   3688:    1. https://www.quora.com/Can-gravitons-escape-the-event-horizon-of-a-black-hole-If-not-how-can-we-detect-the-black-holes-gravity
                   3689:    2. http://physics.stackexchange.com/questions/107185/are-gravitons-bound-by-the-event-horizon
                   3690:    3. http://curious.astro.cornell.edu/physics/89-the-universe/black-holes-and-quasars/theoretical-questions/451-how-do-gravitons-escape-black-holes-to-tell-the-universe-about-their-gravity-advanced
                   3691:    4. https://www.google.ru/?gws_rd=ssl#q=graviton+event+horizon
                   3692: 
                   3693: Автор:
                   3694: Мишель Матвеев (Санкт-Петербург)
                   3695: 
                   3696: Вопрос 10:
                   3697: Согласно несерьезному мнению, это произведение следует признать
                   3698: экстремистским за описание убийства главы государства при помощи дрона.
                   3699: Назовите это произведение.
                   3700: 
                   3701: Ответ:
                   3702: "Сказка о золотом петушке".
                   3703: 
                   3704: Комментарий:
                   3705: Золотой петушок, убивший царя Додона, - вполне себе дрон. А домик Элли,
                   3706: если что, не был управляемым, да и Гингема - не глава государства.
                   3707: 
                   3708: Источник:
                   3709: http://www.anekdot.ru/id/822764/
                   3710: 
                   3711: Автор:
                   3712: Мишель Матвеев (Санкт-Петербург)
                   3713: 
                   3714: Вопрос 11:
                   3715: Одна из версий объясняет происхождение известного спортивного термина
                   3716: так: он образован от французского слова "l'&oelig;uf" [лёф], означающего
                   3717: яйцо, а яйцо похоже на... Что?
                   3718: 
                   3719: Ответ:
                   3720: Ноль.
                   3721: 
                   3722: Комментарий:
                   3723: В теннисе ноль очков почему-то называют словом "love" [лав]. Причины и
                   3724: пытается объяснить приведенная версия.
                   3725: 
                   3726: Источник:
                   3727: http://en.wikipedia.org/wiki/Tennis_scoring_system
                   3728: 
                   3729: Автор:
                   3730: Мишель Матвеев (Санкт-Петербург)
                   3731: 
                   3732: Вопрос 12:
                   3733: В рассказе об известном произведении говорится, что в Карлсбаде некто
                   3734: пошутил: "Видите, как действуют на организм карлсбадские воды!".
                   3735: Назовите это произведение.
                   3736: 
                   3737: Ответ:
                   3738: "Прощальная симфония".
                   3739: 
                   3740: Комментарий:
                   3741: По версии шутника, музыканты один за другим уходили со сцены по личным
                   3742: мотивам. Кстати, исполнялась не сама "Прощальная симфония" Гайдна, а
                   3743: произведение Рихарда Штрауса, использовавшее ту же идею постепенного
                   3744: ухода музыкантов.
                   3745: 
                   3746: Источник:
                   3747: http://www.blagaya.ru/put/articles/muzhumor/
                   3748: 
                   3749: Автор:
                   3750: Мишель Матвеев (Санкт-Петербург)
                   3751: 

FreeBSD-CVSweb <freebsd-cvsweb@FreeBSD.org>